Welcome to
Science a GoGo's
Discussion Forums
Please keep your postings on-topic or they will be moved to a galaxy far, far away.
Your use of this forum indicates your agreement to our terms of use.
So that we remain spam-free, please note that all posts by new users are moderated.


The Forums
General Science Talk        Not-Quite-Science        Climate Change Discussion        Physics Forum        Science Fiction

Who's Online Now
0 members (), 335 guests, and 2 robots.
Key: Admin, Global Mod, Mod
Latest Posts
Top Posters(30 Days)
Previous Thread
Next Thread
Print Thread
Page 1 of 5 1 2 3 4 5
#56184 07/03/16 02:57 PM
Joined: Mar 2006
Posts: 4,136
P
paul Offline OP
Megastar
OP Offline
Megastar
P
Joined: Mar 2006
Posts: 4,136
Im going to request that any reply to this post
be removed by a moderator if the reply is not scientifically
supported or is off topic or misleading or religion oriented
or hateful or slanderous or basically down right terroristic.

in other words if orac or samwik replies. grin


when thinking about sunlight ( naturally generated light )
from our sun not artificially generated light caused by the
manipulation of electric currents through a medium that
produces electromagnetic waves or any other process.

Im thinking that the electromagnetic waves generated by
our sun contain the entire spectrum of electromagnetic waves
that our sun can generate and transmit.

and that the transmission of the entire spectrum of those sun
generated electromagnetic waves are contained in a continuous
wave with frequencies that range from the shortest wave to the longest
wave then from the longest wave to the shortest wave.

like copying the below image and flipping it horizontally
and placing the two images side by side with the two
long waves facing each other.





electromagnetic waves generated by our sun are due to
a continuous process within our sun so the electromagnetic
wave produced by this process must also be continuous.

the reason that we can detect specific wave lengths of
electromagnetic waves is because we have built detection
equipment to detect those specific electromagnetic waves.


in order for our sun to emit or transmit all of the
frequencies of electromagnetic waves that it generates
our sun would need to consist of all of the molecules
that can emit those specific frequencies and all of those
molecules would need to also be present in all of that specific
molecules modes that that specific molecule must be in in
order to absorb or emit a photon with a specific frequency , I highly doubt that this is what is happening.

the suns temperature alone would prevent the above from
occurring.



3/4 inch of dust build up on the moon in 4.527 billion years,LOL and QM is fantasy science.
.
paul #56185 07/03/16 05:12 PM
Joined: Dec 2010
Posts: 1,858
B
Megastar
Offline
Megastar
B
Joined: Dec 2010
Posts: 1,858
Originally Posted By: Paul
electromagnetic waves generated by our sun are due to
a continuous process within our sun so the electromagnetic
wave produced by this process must also be continuous.

Unfortunately your basic premise is wrong. Electromagnetic waves are generated in the sun by the transition of electrons from one state to another. The process is not continuous. One photon is generated for each transition from a higher state to a lower one. The wavelength of each photon is determined by the difference in energy between the higher state and the lower one. There is no continuous source of photons, other than the fact that the Sun creates photons one at a time at such short intervals that there appears to be a continuous creation.

In fact it could be kind of like a ditch digger who works continuously to dig a ditch, but he still does it one shovel full at a time. The Sun has a lot of ditch diggers digging photons one at a time.

Note that this is an extreme simplification. The total process by which the photon is created is very complex, but this, I think, clearly defines the last step in the creation of the photon before it leaves the Sun.

Bill Gill


C is not the speed of light in a vacuum.
C is the universal speed limit.
Bill #56186 07/04/16 12:55 AM
Joined: Mar 2006
Posts: 4,136
P
paul Offline OP
Megastar
OP Offline
Megastar
P
Joined: Mar 2006
Posts: 4,136
Bill

would you consider that the ditch digger would be
continually digging the ditch if he lifted the shovel
full of dirt to throw it outside the ditch?

a continuous process that is comprised of a chain of necessary
events or sub processes or movements and actions as
in sticking the shovel into the ground to fill the shovel with
dirt , then lifting the shovel and throwing the dirt
out of the ditch would involve billions upon billions
of sub processes by the ditch diggers brain and nervous
system and muscles and his entire body in general
but can be called a continuous process to get the dirt out
of the ditch.

as in saying he is a ditch digger.

or he performs the continuous process of ditch digging.

like when our sun performs a continuous process of the many
events that lead up to the emission of a photon.

or it performs the continuous process of photon emission.

for instance a continuous process being a chain of events that occur in order to accomplish a goal or that lead up to a event.

have you ever seen a photon detection read out of the sun
that shows a break in the detection stream that it is
detecting and recording that could be described as being
a space between photons?

if there is a break in the stream that is detected would it be
valid to claim it as being a break or would it be more truthful
to admit that the break may simply be of such a low intensity
that the detection equipment might not be capable of
detecting it or picking it up.



so do you think that the sun only emits a photon in one frequency.

or do you think that the sun only emits a photon consisting
of all frequencies that it can emit.

or do you think that the sun emits photons of various frequencies.

I would like your opinion on this because your the EE here
I think from previous post.



3/4 inch of dust build up on the moon in 4.527 billion years,LOL and QM is fantasy science.
Bill #56187 07/04/16 01:14 AM
Joined: Mar 2006
Posts: 4,136
P
paul Offline OP
Megastar
OP Offline
Megastar
P
Joined: Mar 2006
Posts: 4,136
Quote:
The wavelength of each photon is determined by the difference in energy between the higher state and the lower one.


I didnt see this the first time I read it , should I take
it that you do think that the sun emits photons of various frequencies?

in other words the sun does not emit one frequency of a wave.

and that the one wave is not short wave only?







3/4 inch of dust build up on the moon in 4.527 billion years,LOL and QM is fantasy science.
paul #56188 07/04/16 01:56 AM
Joined: Dec 2010
Posts: 1,858
B
Megastar
Offline
Megastar
B
Joined: Dec 2010
Posts: 1,858
As I was attempting to explain, but you don't seem to be interested in understanding, the processes which drive the Sun emit one photon at a time. Throughout its bulk there are many such processes. Each process emits one photon. The frequency of that photon depends on the details of the particular process which leads to that one photon. Since there are a huge number of such processes taking place it appears to a casual viewer that the production of photons is continuous, and that the sun emits the photons in a continuous spread of frequencies. However, careful investigation shows that this is a naive idea. The realization that the photons come one at a time at discrete frequencies has been arrived at through careful observation and experiment over a long period of time. The thing about it is that this is the only explanation that explains all of the observations.

The thing about science is that unless an explanation (theory) takes care of all of the observations then it is not correct. Over several hundreds of years the observation of the way light works can only be accounted for by the above description. As I said of course the actual way it works is much more complex. That is true because the whole thing has to hang together. If just one of the assumptions is wrong the whole thing falls apart. So far no scientist has been able to find anything wrong with the theories that lead to this description. Whenever somebody says that it is wrong they have not studied the facts closely enough to detect the problems with their ideas.

Bill Gill


C is not the speed of light in a vacuum.
C is the universal speed limit.
Bill #56189 07/04/16 02:17 AM
Joined: Mar 2006
Posts: 4,136
P
paul Offline OP
Megastar
OP Offline
Megastar
P
Joined: Mar 2006
Posts: 4,136
ok , I get it.

so the photons that are emitted from our sun must
be emitted by the elements that make up the mass of the
surface or photosphere of the sun.

Photospheric composition of our sun (by mass)

Hydrogen 73.46%
Helium 24.85%
Oxygen 0.77%
Carbon 0.29%
Iron 0.16%
Neon 0.12%
Nitrogen 0.09%
Silicon 0.07%
Magnesium 0.05%
Sulfur 0.04%


this must mean that the photons must be emitted by
some or even all of the above 10 elements that make
up our suns photosphere.

nitrogen cannot absorb a photon nor can oxygen
as far as I understand and I will look up the rest.

so the em spectrum that the sun emits must come from
the above elements.



3/4 inch of dust build up on the moon in 4.527 billion years,LOL and QM is fantasy science.
paul #56190 07/04/16 04:29 AM
Joined: May 2011
Posts: 2,819
O
Megastar
Offline
Megastar
O
Joined: May 2011
Posts: 2,819
Originally Posted By: Bill G
The wavelength of each photon is determined by the difference in energy between the higher state and the lower one.

Bill G you have dug yourself into a hole with that answer. Whilst the statement is true that is only a minor part of the answer.

The logic from your statement follows the sun can only emit the spectra of the various elements and the emission should be in very distinct frequencies of the elements perhaps a few hundred spectral lines at most.

It's quite clear the answer is wrong as the suns emission is quite broad from UV down thru Visible spectrum to high IR and includes most frequencies and the emission has no real peaks in particular frequencies.

You need to do some reading on the sun, start with blackbody. Like I said to Samwik in the other post, trying to fix Paul's errors, while making errors just as bad does not help.

I would suggest you just leave Paul alone with this unless you are prepared to do a lot of reading Bill because there are more than a few gotchas and your answer is likely to be as bad as his. Sorry Bill, I am really skeptical you will be able to give the correct scientific answer just based on science discussions with you. For myself, I am unwilling to get involved in yet another stupid argument with Paul, so I suggest we all just let him go with the thread like we do Marosz.

Possibly you could start with an old filament globe for your understanding but there are still a lot of other things to add in. I am sure you see a filament (incandescent) light doesn't work in the way you described above.

I know you mean well and like science, but this is one of those cases we discussed. You need to either drop out or completely rethink your answer remembering the end result is a rather smooth broadband spectrum of emission frequencies.

Last edited by Orac; 07/04/16 08:14 AM.

I believe in "Evil, Bad, Ungodly fantasy science and maths", so I am undoubtedly wrong to you.
Orac #56191 07/04/16 01:17 PM
Joined: Dec 2010
Posts: 1,858
B
Megastar
Offline
Megastar
B
Joined: Dec 2010
Posts: 1,858
Originally Posted By: Orac
Bill G you have dug yourself into a hole with that answer. Whilst the statement is true that is only a minor part of the answer.

The logic from your statement follows the sun can only emit the spectra of the various elements and the emission should be in very distinct frequencies of the elements perhaps a few hundred spectral lines at most.

Orac, you are at it again. I didn't say any of that. I said it was very complex. The only statement I made was that the photons are created when an electron drops from 1 energy state to a lower one. The frequency depends on the difference in the energy. That says nothing about where the electron is. All that is required is that there be a change in energy. I didn't say anything about 'peaks in frequency'. I was trying to point out to Paul that he was wrong about the process that leads to radiation from the Sun is continuous.

You are trying to put a lot of stuff into my words which I specifically excluded to get down to the very basic topic that I was addressing. I was not trying to define any of the processes that occur. If you can figure a way for the Sun to emit photons which does not contain energy transitions please let me know.

Taking exception to somebody who is trying to teach a first grader how physics works by simplifying the subject is not a great idea. It just make you look like a jerk.

Bill Gill


C is not the speed of light in a vacuum.
C is the universal speed limit.
Bill #56192 07/04/16 01:49 PM
Joined: Mar 2006
Posts: 4,136
P
paul Offline OP
Megastar
OP Offline
Megastar
P
Joined: Mar 2006
Posts: 4,136
Bill

Quote:
The logic from your statement follows the sun can only emit the spectra of the various elements and the emission should be in very distinct frequencies of the elements perhaps a few hundred spectral lines at most.

It's quite clear the answer is wrong as the suns emission is quite broad from UV down thru Visible spectrum to high IR and includes most frequencies and the emission has no real peaks in particular frequencies.


so its like at any time (t).

t=1 : the sun emits a broad range of photons like a
conveyor belt full of individual items and each item
is different from all of the other items and these items
make up the electromagnetic spectrum of the sun.

t=2 : the sun emits a broad range of photons like a
conveyor belt full of individual items and each item
is different from all of the other items and these items
make up the electromagnetic spectrum of the sun.

t=3 : the sun emits a broad range of photons like a
conveyor belt full of individual items and each item
is different from all of the other items and these items
make up the electromagnetic spectrum of the sun.

so that an object on the earth that is exposed to sunlight
will heat up evenly due to its color.

suppose there were a
1 mile square area glass mirror on the earths surface
that is painted black.

directly next to it there is a
1 mile square area glass mirror on the earths surface
that is painted white.

the heat that the two areas absorb due to exposure to the
suns light will be evenly spread out on each surface
yet the two areas will absorb different amounts of heat.

if I have photon detection equipment in the center of each
of the two areas the equipment will not detect a significant difference
in the spectrum of light that is reaching the equipment.

nor will it detect any significant peaks in the number of
any of the waves that make up the spectrum.

although the entire spectrum of light is detected by the
surface of each area , the only portion of the spectrum that
is absorbed by the paint on each surface is the portion
of the spectrum of light that the color of paint can absorb
and the remainder of the spectrum is reflected by the paint.

I personally doubt that the sun is emitting an exact
number of individual photons that all have different
wavelengths at any point in time.

this would require that all elements involved in
any process that could generate light from the
sun would need to be evenly distributed at every
point of photon emission.

this also means that each and every point on the suns surface
must emit photons in all frequencies and in all directions
due to the simple fact that light travels in a straight line.

that process would be extremely impossible

and this is a few of the reasons why I think that light
from our sun is radiated in a continuous stream like I
posted in the OP.

but there may be an answer found in further study that
shows that I am wrong about it...




3/4 inch of dust build up on the moon in 4.527 billion years,LOL and QM is fantasy science.
Bill #56193 07/04/16 02:09 PM
Joined: May 2011
Posts: 2,819
O
Megastar
Offline
Megastar
O
Joined: May 2011
Posts: 2,819
Originally Posted By: Bill
Taking exception to somebody who is trying to teach a first grader how physics works by simplifying the subject is not a great idea. It just make you look like a jerk

Sorry you are the one who looks like a jerk because it's appears you don't actually know the answer and now trying to blame me.

Simple question how and why is the spectrum broad and smooth? Prove to me you know the answer because that is what Paul is wondering.

Hint the answer HAS NOTHING TO DO WITH TRANSITION OF ELECTRON ENERGY LEVELS which produce distinct precise frequencies
Originally Posted By: Bill G
Electromagnetic waves are generated in the sun by the transition of electrons from one state to another.

So why don't you man up and take my advice and start with an incandescent globe.

You don't need energy levels or any such junk to understand it ... just think about it ... not like you haven't seen an incandescent globe is it. You claim you are trying to simplify the situation SO GO AHEAD AND SIMPLIFY THE PROBLEM and stop complaining about me.

Last edited by Orac; 07/04/16 02:44 PM.

I believe in "Evil, Bad, Ungodly fantasy science and maths", so I am undoubtedly wrong to you.
Orac #56194 07/04/16 02:49 PM
Joined: May 2011
Posts: 2,819
O
Megastar
Offline
Megastar
O
Joined: May 2011
Posts: 2,819
Paul this is a one time post I will not discuss it but I will give you the true scientific answer.

https://en.wikipedia.org/wiki/Thermal_radiation
Quote:
Thermal radiation is electromagnetic radiation generated by the thermal motion of charged particles in matter. All matter with a temperature greater than absolute zero emits thermal radiation. When the temperature of the body is greater than absolute zero, inter-atomic collisions cause the kinetic energy of the atoms or molecules to change. This results in charge-acceleration and/or dipole oscillation which produces electromagnetic radiation, and the wide spectrum of radiation reflects the wide spectrum of energies and accelerations that occur even at a single temperature.

So the official science line is it is the movement of charged particles induced by temperature vibration that creates such a wide smooth spectrum.

An incandescent lamp works the same way which is why the filament has to be white hot.

It has nothing to do with electrons and energy levels which are ionization things and why I took exception to Bill G's answer. So being clear the energy levels in atoms etc are not in play in the effect, there is however limited ionization withing elements in the sun but its a tiny percentage of the emission. The bulk of the suns radiation is thermal which is about kinetic energy of particles not energy levels of anything.

You are correct in that the process does not require or dictate that the emissions are photons, that evidence comes from analysis of the sunlight. The suns emissions contain Shot noise or Poisson noise which can only come about by having discrete packets of energy. So that is if you like direct observation that sun emissions are indeed photons.

Why a smooth kinetic process still ends up putting out photons I will not answer because it requires a depth of science you don't yet have and frankly won't accept anyhow. I suspect you will just ignore all the evidence the emission has photons because that is a bit of an issue for you.

Lets just say in layman terms the suns spectrum is because the sun is hot ... can't be any simpler.

No energy levels were required in the making of this post ... I did however need Kinetic Energy, Bill smile

So Paul you were closer to actual science than Bill G and why I called it so ... I don't play favourites.

FEEL FREE TO ARGUE AGAINST IT ... I WILL NOT RESPOND ... I WILL LEAVE YOU TOO IT.

Last edited by Orac; 07/04/16 03:32 PM.

I believe in "Evil, Bad, Ungodly fantasy science and maths", so I am undoubtedly wrong to you.
Orac #56195 07/04/16 03:31 PM
Joined: Mar 2006
Posts: 4,136
P
paul Offline OP
Megastar
OP Offline
Megastar
P
Joined: Mar 2006
Posts: 4,136
So my OP is correct.

the thermal energy that the sun radiates is a combination
of the heat that is generated by the continuous processes
that occur within the sun.

also the heat that a process generates does not appear in
its highest magnatude suddenly ...

the thermal heat continuously builds and then falls in magnitude.

and it is the continuous building and falling of the
radiated thermal energy that results in what we detect
as the electromagnetic spectrum.

Quote:
and that the transmission of the entire spectrum of those sun
generated electromagnetic waves are contained in a continuous
wave with frequencies that range from the shortest wave to the longest
wave then from the longest wave to the shortest wave.

like copying the below image and flipping it horizontally
and placing the two images side by side with the two
long waves facing each other.








3/4 inch of dust build up on the moon in 4.527 billion years,LOL and QM is fantasy science.
paul #56196 07/04/16 03:33 PM
Joined: May 2011
Posts: 2,819
O
Megastar
Offline
Megastar
O
Joined: May 2011
Posts: 2,819
Originally Posted By: paul
So my OP is correct

Not exact but yes closer than Bill G's answer, I would have accepted it. That is why I picked on Bill G not you.

As stated the problem is the emission when measured definitely contains photons which will perplex your answer.

I wouldn't for example use "continuous building and falling" the charged particles are just moving and colliding randomly the emission reflects that. You correctly got it as heat but missed converting that to kinetic energy.

Your question is how and why does the kinetic energy get quantized in the collisions. There is no obvious answer for you, it's not what you see in classical physic collisions. Which is why I suspect you will try to deny the fact rather than deal with the fact and why I am staying out of this discussion.

For all that I am never right smile So don't use me as a reference ... but you now have the terms and a start reference.

I am sorry I had to interject, but you should see why, I hate butchered science .... I will now leave you both with it.

Last edited by Orac; 07/04/16 04:31 PM.

I believe in "Evil, Bad, Ungodly fantasy science and maths", so I am undoubtedly wrong to you.
Orac #56197 07/04/16 08:02 PM
Joined: Dec 2010
Posts: 1,858
B
Megastar
Offline
Megastar
B
Joined: Dec 2010
Posts: 1,858
Originally Posted By: Orac
Sorry you are the one who looks like a jerk because it's appears you don't actually know the answer and now trying to blame me.

Simple question how and why is the spectrum broad and smooth? Prove to me you know the answer because that is what Paul is wondering.

That isn't what I saw Paul asking. He was making a statement that
Originally Posted By: Paul

and that the transmission of the entire spectrum of those sun
generated electromagnetic waves are contained in a continuous
wave with frequencies that range from the shortest wave to the longest
wave then from the longest wave to the shortest wave.

This to means that he thinks that all of the frequencies are in one wave. That suggests that the frequencies are all in one container, so to speak. What I was telling him is that each photon is a separate entity.

Beyond that I had nothing to say. I was not at all discussing the fact that there is a range of frequencies that are emitted by the Sun. And I was not discussing why the shape of the spectrum of those frequencies is what it is. I was specifically discussing the emission of one photon.

If you weren't in such a hurry to make things more complex than what they need to be then you would not jump on what other people say and try to show how much smarter you are than they are.

I stand by my statements, because I was trying to keep it down to a level that a first grader could understand.

I just went back and looked at the answer following the one I wrote this in response to. You say it is thermal, so it is not a change in energy levels. Well, if you can create a photon without changing energy levels tell me how. I don't think that you can create a photon without having a charged particle change energy levels. Photon are electromagnetic particles/fields produced by the motion of charged particles.

Bill Gill

Last edited by Bill; 07/04/16 08:11 PM.

C is not the speed of light in a vacuum.
C is the universal speed limit.
Bill #56198 07/04/16 10:52 PM
Joined: Oct 2006
Posts: 1,164
Megastar
Offline
Megastar
Joined: Oct 2006
Posts: 1,164
Originally Posted By: Bill
....The only statement I made was that the photons are created when an electron drops from 1 energy state to a lower one. The frequency depends on the difference in the energy. That says nothing about where the electron is. All that is required is that there be a change in energy.
Wouldn’t nuclear fusion create most of the photons in the sun?
I would expect, at the sun’s temperatures, most atoms would be stripped of their electrons …so electronic transitions wouldn’t be that common.

I posted a link from NASA about how photons take many multiple millennia to travel from the core of the sun to its surface.

They note how “Originally born as energetic gamma rays, after billions of collisions with matter, this radiation [as photons] reaches the surface and escapes into space.”

I would imagine, during those “billions of collisions,” that most of the original gamma rays would be “spread out” or “shifted” down into lower energy frequencies, thus (perhaps?) generating the full solar spectrum.

But that is just what I expect and imagine ...and guess. I'd be happy to see any sources citing otherwise. wink
~

p.s. I’m guessing that some large-scale mechanical processes such as convection, possibly involving magnetic fields, might also produce photons of their own; though ultimately these processes are driven by the forces of nuclear fusion and gravity too, so they still depend on the original production of gamma rays.


Pyrolysis creates reduced carbon! ...Time for the next step in our evolutionary symbiosis with fire.
samwik #56199 07/05/16 02:48 AM
Joined: May 2011
Posts: 2,819
O
Megastar
Offline
Megastar
O
Joined: May 2011
Posts: 2,819
Actually you are pretty dam spot on Sam ... the nuclear reactions involving energy level reactions simply create the heat. The heat creates the light from the kinetic energy of the heat which is why is why the spectrum is smooth and broad .... which is the problem Paul was after in the OP.

Last edited by Orac; 07/05/16 03:41 AM.

I believe in "Evil, Bad, Ungodly fantasy science and maths", so I am undoubtedly wrong to you.
Bill #56200 07/05/16 03:02 AM
Joined: May 2011
Posts: 2,819
O
Megastar
Offline
Megastar
O
Joined: May 2011
Posts: 2,819
Originally Posted By: Bill
This to means that he thinks that all of the frequencies are in one wave. That suggests that the frequencies are all in one container, so to speak. What I was telling him is that each photon is a separate entity.

And he is correct Bill G there simply is no reason within the creation process as to why photons rather than waves should be created ... I said that.

It is the same situation on an incandescent bulb there simply is no reason in classical physics that the light emitted is photons rather than waves. You have kinetic energy and that usually isn't quantized.

You keep saying I am making it more complex I am making it DEAD SIMPLE ... HEAT THAT MAKES THE LIGHT. Why that light is quantized into photons you can't get from the process. Most children get that and then also understand why fire makes light etc.

Originally Posted By: Bill G
I don't think that you can create a photon without having a charged particle change energy levels.

There is the physics you are missing and why I took you to task smile

The connection between kinetic energy of molecules and emission comes from the photoelectric effect which Einstein won a nobel prize for nothing to do with energy levels in the particles. The energy level quantization is in the EM NOT THE PARTICLES.

https://en.wikipedia.org/wiki/Photoelectric_effect
Quote:
Photoemission can occur from any material, but it is most easily observable from metals or other conductors because the process produces a charge imbalance, and if this charge imbalance is not neutralized by current flow (enabled by conductivity), the potential barrier to emission increases until the emission current ceases. It is also usual to have the emitting surface in a vacuum, since gases impede the flow of photoelectrons and make them difficult to observe. Additionally, the energy barrier to photoemission is usually increased by thin oxide layers on metal surfaces if the metal has been exposed to oxygen, so most practical experiments and devices based on the photoelectric effect use clean metal surfaces in a vacuum.

You then connect it thru Max Plancks and black body radiation to get the complete story and he derives the formula E=hv and the energy must be quantized. The quantization is not produced by the process it is because all electromagnetic waves must be quantized.

Originally Posted By: Max Planck
Planck postulate, that electromagnetic energy could be emitted only in quantized form, in other words, the energy could only be a multiple of an elementary unit

ALL EM is quantized ... ALL NO EXCEPTIONS.

This allows all EM to have "photons" which is the packet multiples of the minimum level of quantization. Classically you don't think of RF for example as being able to have photons you make it a wave in classical electromagnetics. I suspect that is why you are trying to make the process create the quantization and photons.

Why a photon is emitted is because the kinetic energy (motion) of a charged particle changed/reveresed and it gives up an energy packet quantized to that energy amount. The quantization is in the EM not the process which is actually linear.

The upshot is every material emits the same color at the same temperature telling you the emission has nothing to do with ENERGY LEVELS WITHIN THE MATERIAL. White hot is white hot and is the same temperature regardless of the material.

ANY ANSWER THAT INVOLVES PROPERTIES OF THE MATERIAL IS WRONG.

Last edited by Orac; 07/05/16 07:08 AM.

I believe in "Evil, Bad, Ungodly fantasy science and maths", so I am undoubtedly wrong to you.
samwik #56201 07/05/16 03:52 AM
Joined: Mar 2006
Posts: 4,136
P
paul Offline OP
Megastar
OP Offline
Megastar
P
Joined: Mar 2006
Posts: 4,136
https://solarsystem.nasa.gov/docs/Photons-ST-PO.pdf

The Convection Zone: The Mystery Layer

When the photons arrive at the convection layer, 150,000 km
below the sun’s surface, the nuclei are able to hold on to
electrons, and neutral atoms and ions are formed.
And photon
energies have been degraded to the point that gaseous
atoms and ions absorb the energy of the photons and hold it,
rather than having it bounce off
(or be absorbed and re radiated).
These atoms effectively block the outward flow of radiative
energy and the energy absorbed by the atoms
makes them enormously hot
.

At that point the convection currents take over and carry
the sun’s energy to the photosphere on seething rivers of hot
gases.
Although it may have taken the photons a million years
to reach the convection zone, the energy they deliver
rises through the entire convention zone in about three months.
All the energy emitted at the surface of the sun is
transported there by convection.

The Photosphere: Ah! Light At Last!

The photosphere, at the top of the convection zone, is the
visible bright surface of the sun.
Here the gaseous atoms no longer block radiative flow.
As the hot atoms cool, they release their excess energy once
again as photons
that stream unimpeded into space and ultimately
provide support for life on Earth


that makes sense.


as the hot atoms cool...

photons are emitted because of a continuous energy level
change from higher energy levels to lower energy levels.

and this process streams the spectrum.

not sure what happens to the atoms after they reach a
certain energy level..!!!


3/4 inch of dust build up on the moon in 4.527 billion years,LOL and QM is fantasy science.
paul #56202 07/05/16 03:57 AM
Joined: May 2011
Posts: 2,819
O
Megastar
Offline
Megastar
O
Joined: May 2011
Posts: 2,819
Agreed Paul the heat makes the photons I pretty much think I covered all that and you were close its a fairly continual process. You notice they just convert the heat to photons not waves .. as trying to explain to Bill G that requires a more complex explaination from different lines of evidence.

In some ways they did a bit of hand wave to create the photons at the end and don't explain why.

The question you may ponder is does heat always release photons can it ever release waves?

Last edited by Orac; 07/05/16 04:22 AM.

I believe in "Evil, Bad, Ungodly fantasy science and maths", so I am undoubtedly wrong to you.
Orac #56203 07/05/16 04:22 AM
Joined: Mar 2006
Posts: 4,136
P
paul Offline OP
Megastar
OP Offline
Megastar
P
Joined: Mar 2006
Posts: 4,136
none of this process is proven , we really dont know exactly
how it works.

so the only real thing we have to work with is what we can
detect with what we have built to detect with.

at the end of the day its just a vibration.

its not what we think of as a photon or a wave
its a expanding vibrating sphere.








3/4 inch of dust build up on the moon in 4.527 billion years,LOL and QM is fantasy science.
paul #56204 07/05/16 04:24 AM
Joined: May 2011
Posts: 2,819
O
Megastar
Offline
Megastar
O
Joined: May 2011
Posts: 2,819
The problem comes when you observe one of those spheres ... but we have done that story ... and I will now leave you smile

Calling them a wobbly sphere doesn't change what they are, any more than calling them a photon it's just a name smile

We covered that story to death before and I have corrected all the butchered science and you have something close to what science actually says. So a good time to butt out.

Last edited by Orac; 07/05/16 04:50 AM.

I believe in "Evil, Bad, Ungodly fantasy science and maths", so I am undoubtedly wrong to you.
Orac #56205 07/05/16 04:52 AM
Joined: Mar 2006
Posts: 4,136
P
paul Offline OP
Megastar
OP Offline
Megastar
P
Joined: Mar 2006
Posts: 4,136
Im not saying it is a problem ...

but you are talking about a single artificially generated
photon.

not the naturally generated spectrum of what we call light
from our sun.

we dont see light our eyes detect vibrations.

when a object heats up its atoms are simply vibrating more.

anything that is today called a light wave or a photon of light simply causes objects to vibrate.





3/4 inch of dust build up on the moon in 4.527 billion years,LOL and QM is fantasy science.
paul #56206 07/05/16 05:07 AM
Joined: Sep 2012
Posts: 1,209
N
Megastar
Offline
Megastar
N
Joined: Sep 2012
Posts: 1,209
How motion is chaging SUN's light


for example we have BULB ( yellow light )
satellite is accelerating accelerating .....

How motion changing Yellow colour ? ( colour is constant ? )
How big force is pushing SAIL ? ( constant ? )




How big force is pushing EARTH ? constant ( the same ? )


How works my eyes ( ideal support or elatic plasticine ? )


Photon >>> hot platicine

How many energy photon will lost ?

exist smaller energy's portion than photon ? ( if photon lost energy soo it schould exist lower energy portion than photon !)

Last edited by newton; 07/05/16 05:08 AM.
paul #56207 07/05/16 05:08 AM
Joined: May 2011
Posts: 2,819
O
Megastar
Offline
Megastar
O
Joined: May 2011
Posts: 2,819
Originally Posted By: paul
anything that is today called a light wave or a photon of light simply causes objects to vibrate.

That is actually scary close even though I don't think you meant it to be laugh

Marosz is here to help you now Paul, so catch you later. To much plasticine and crayons for me, I am definitely exiting stage right ===========>

Last edited by Orac; 07/05/16 05:14 AM.

I believe in "Evil, Bad, Ungodly fantasy science and maths", so I am undoubtedly wrong to you.
Orac #56208 07/05/16 05:19 AM
Joined: Mar 2006
Posts: 4,136
P
paul Offline OP
Megastar
OP Offline
Megastar
P
Joined: Mar 2006
Posts: 4,136
why would that be scary?

its exactly what happens.

its much easier to describe light as a vibration than
as a photon or a wave.

action and reaction.

a portion of the light spectrum is the same frequency of
most or any atom.

so simple ...


3/4 inch of dust build up on the moon in 4.527 billion years,LOL and QM is fantasy science.
Orac #56209 07/05/16 05:24 AM
Joined: Sep 2012
Posts: 1,209
N
Megastar
Offline
Megastar
N
Joined: Sep 2012
Posts: 1,209
photon >>> .......plasticine (object )

how big force is pushing stationary plasticine
and how big force is pushing not stationary plasticyne ?


photon >>> .......30km/s <<<plasticine


photon >>> .......plasticine >>> 30 km/s


HOW BRIGHT PICTURE IS REGISTERING CAMERA ???






RELATIVE or ABSOLUTE ?

m >>>V........M

OR

m..........V<<< M

OR

m >>>1/2V .....1/2V<<< M

do we able measure deformation ?

[img]https://2.bp.blogspot.com/-QvRGmH4z8MU/V...cB/s1600/22.bmp[/img]

are we able recognize that big mass M is moving or small mass m is moving ( how will look collision ? )


Big Car >>>V .....small car

small car >>>V....Big car






Last edited by newton; 07/05/16 05:27 AM.
paul #56210 07/05/16 05:24 AM
Joined: May 2011
Posts: 2,819
O
Megastar
Offline
Megastar
O
Joined: May 2011
Posts: 2,819
Originally Posted By: Paul
why would that be scary?

You just described the foundations of QM which is built on that exact premise and where Max Planck started. His logic was exactly the same as yours we only called it photons later.

You will now instantly retract that because you can't believe in anything to do with QM laugh

Last edited by Orac; 07/05/16 05:26 AM.

I believe in "Evil, Bad, Ungodly fantasy science and maths", so I am undoubtedly wrong to you.
Orac #56211 07/05/16 05:33 AM
Joined: Mar 2006
Posts: 4,136
P
paul Offline OP
Megastar
OP Offline
Megastar
P
Joined: Mar 2006
Posts: 4,136
he must have been a pretty smart guy then...

I suppose that is scary and perhaps I should have waited
until halloween. sick

that doesnt mean however that I agree with QM



3/4 inch of dust build up on the moon in 4.527 billion years,LOL and QM is fantasy science.
paul #56212 07/05/16 05:44 AM
Joined: May 2011
Posts: 2,819
O
Megastar
Offline
Megastar
O
Joined: May 2011
Posts: 2,819
Originally Posted By: paul
he must have been a pretty smart guy then...

I suppose that is scary and perhaps I should have waited
until halloween. sick

that doesnt mean however that I agree with QM

Yes,he was a very smart guy and I don't claim you believe in QM.

Planck is considered the father of Quantum mechanics for exactly that thought. He started conceptually like you did with the vibrations as a concept, what he discovers is the vibrations are real and because of that they have "discrete energy amounts" as they are a vibration.

Originally Posted By: Max Planck history . wikipedia
At first Planck considered that quantisation was only "a purely formal assumption ... actually I did not think much about it...";

What it does say is you agree with it's QM's birth premise. You think that QM theory fails somewhere higher up.

There is however a massive sting in the tail of that premise smile

Last edited by Orac; 07/05/16 06:07 AM.

I believe in "Evil, Bad, Ungodly fantasy science and maths", so I am undoubtedly wrong to you.
paul #56214 07/05/16 01:33 PM
Joined: Dec 2010
Posts: 1,858
B
Megastar
Offline
Megastar
B
Joined: Dec 2010
Posts: 1,858
Orac, Since you have now come around to the fact that QM is not what runs the sun, it is run by classic physics, I guess I will just go back to my policy of ignoring you.

Bill Gill


C is not the speed of light in a vacuum.
C is the universal speed limit.
Orac #56215 07/05/16 02:28 PM
Joined: Mar 2006
Posts: 4,136
P
paul Offline OP
Megastar
OP Offline
Megastar
P
Joined: Mar 2006
Posts: 4,136
Quote:
Yes,he was a very smart guy and I don't claim you believe in QM.


believe it or not this has spurned an interest that I
now need to explore.

Im going to see if I can find some videos on Max
we may have a lot more in common than just that simple thought.

I may have inadvertently added Max as a soul mate so I need
to check and see if he will be compatible with Issac Albert
and Carl to name a few.

it looks like we will all just get along just fine.

https://withalliamgod.wordpress.com/2010/11/28/max-planck-on-god/

Quote:
As a physicist, that is, a man who had devoted his whole life to a wholly prosaic science, the exploration of matter, no one would surely suspect me of being a fantast. And so, having studied the atom, I am telling you that there is no matter as such! All matter arises and persists only due to a force that causes the atomic particles to vibrate, holding them together in the tiniest of solar systems, the atom

Yet in the whole of the universe there is no force that is either intelligent or eternal, and we must therefore assume that behind this force there is a conscious, intelligent Mind or Spirit. This is the very origin of all matter.



I see that I just violated my own rules for this thread
by posting content that is religion oriented , so if anyone
would like for me to delete the religion oriented content
just say so and I will.

but you need to say so before your time expires.









3/4 inch of dust build up on the moon in 4.527 billion years,LOL and QM is fantasy science.
Bill #56216 07/05/16 02:40 PM
Joined: May 2011
Posts: 2,819
O
Megastar
Offline
Megastar
O
Joined: May 2011
Posts: 2,819
Bill G, Do as you like, you seem to think I should care what you do, say or believe. Hell I didn't even know I was being ignored so that is how much I give a rats about you.

Let me straighten one bit however, the sun runs on QM it just doesn't involve energy levels of the material in the sun which seemed to be your claim.

White hot is white hot whether the material is hydrogen, iron, lava on earth, Bill G's head or any other material and the emission is the same because the process doesn't rely on anything to do with the material that is being heated.

The connection is between heat and electromagnetic waves NOT THE MATERIAL THAT IS BEING HEATED ... that is what actual science says. That EM waves being quantized can only accept the kinetic heat energy in quanta.

You wanted simple that is layman simple most 12 year old children can get that when told.

Whether you believe it or not I couldn't give a dam about. Go ask it on any physics forum you like and to any scientists you like.

If you can't handle that then please ignore me because I have no intention of believing anything else anytime soon, unless you have new data the science community hasn't seen.

Now good bye or discuss Bill G, I personally don't give a rats.

Last edited by Orac; 07/05/16 02:49 PM.

I believe in "Evil, Bad, Ungodly fantasy science and maths", so I am undoubtedly wrong to you.
paul #56217 07/05/16 02:54 PM
Joined: May 2011
Posts: 2,819
O
Megastar
Offline
Megastar
O
Joined: May 2011
Posts: 2,819
Originally Posted By: paul
believe it or not this has spurned an interest that I
now need to explore.

No worries his biography gives a brief outline of him.

http://www.nobelprize.org/nobel_prizes/physics/laureates/1918/planck-bio.html

You will see he was lead to the problem you are dealing with when he was studying radiation. The question of why is the suns spectrum broad and smooth.

He formalized your vibrations into a resonant element and when you do so the energy exchanges can only be in discrete packet or quanta.

Quote:
revolutionary idea that the energy emitted by a resonator could only take on discrete values or quanta. The energy for a resonator of frequency v is hv where h is a universal constant, now called Planck's constant.

So when your vibrating molecules try to unload their kinetic energy they can only do so in discrete packets and that is the origin of photons but having a broad frequency range representing the broad range of kinetic energies in the molecules.

In effect old blacksmiths already knew this because they had worked white hot metal and the composition of the metal didn't change the color it always has the same distinctive color.

This is the emission curve you get using the theory for different temperatures 5000K, 4000K, 3000K

So heat any material to 5000K and you get visible light and that explains the incandescent light bulb.

When you look at molten lava you can tell the temperature just by looking at the color you don't have to know the composition at all.

Compare your suns emission graph to the 5000K curve developed by Max from theory smile

Here lets make in simple and put on the same post:


Shame I can't get them scaled the same ... but do you see the two graphs match (your image has the 5250 deg C planck graph in black). I can tell the sun is around 5000K in temperature just from the graph. What I don't have is a clue what the sun is made of that requires a different technique and different data. Our theory says it doesn't matter what the material is the heat of 5000K will produce that curve.

So Max was the first person that could explain that emission curve of the sun and it's profound implications. Yes he was religious and we don't hold that against him. His spiritual vibrations are the foundations of QM.

Anyhow that is the scientific origin of that emission curve for the sun, which I and almost all scientists believe. I think Bill G believes something else but he can argue that.

Last edited by Orac; 07/05/16 03:50 PM.

I believe in "Evil, Bad, Ungodly fantasy science and maths", so I am undoubtedly wrong to you.
Orac #56218 07/05/16 04:16 PM
Joined: Mar 2006
Posts: 4,136
P
paul Offline OP
Megastar
OP Offline
Megastar
P
Joined: Mar 2006
Posts: 4,136
Quote:
his derivation of the relationship: this was based on the revolutionary idea that the energy emitted by a resonator could only take on discrete values or quanta. The energy for a resonator of frequency v is hv where h is a universal constant, now called Planck's constant.


Im considering the above

and also seeing this

Quote:
Among these applications and developments may be mentioned Einstein's explanation of the photoelectric effect.


and I will look into Max's universal constant later today.

a foundation is what supports everything above it.
you must be certain that the foundation is solid and
can withstand all criticism .

it says that the energy of a resonated frequency is equal to
e = hv and that h is a "universal" constant.

I think I have a problem with that , I need to check out
how he constructed h

in other words did he include temperature as
a variable element in the construction of "h".

as the temperature always varies and is not a universal
constant in the cosmos when considering the temperatures
radiated from our sun and other suns in the cosmos and that
all frequencies are affected by external forces acting on
them.

he may have included all of that in the construction of "h"
or he may have been focusing on more local heat from our sun.

that will give me something to do later anyway.


3/4 inch of dust build up on the moon in 4.527 billion years,LOL and QM is fantasy science.
paul #56219 07/05/16 04:25 PM
Joined: May 2011
Posts: 2,819
O
Megastar
Offline
Megastar
O
Joined: May 2011
Posts: 2,819
Yes Einstein's photoelectric effect is connected. There is no relativity involved Einstein did other stuff besides relativity smile

It connects heat and light to atoms, that is why does an atom only emit and absorb limited specific frequencies in the ionization process. Einstein used Plancks formula to derive that answer which was also experimentally confirmed. It also explains some important features of atoms.

So now Max's spiritual particles had solved two of the biggest mysteries of early 1900's science.

Last edited by Orac; 07/05/16 04:31 PM.

I believe in "Evil, Bad, Ungodly fantasy science and maths", so I am undoubtedly wrong to you.
Orac #56220 07/05/16 04:31 PM
Joined: Mar 2006
Posts: 4,136
P
paul Offline OP
Megastar
OP Offline
Megastar
P
Joined: Mar 2006
Posts: 4,136
Quote:
but do you see the two graphs match


Im only seeing one graph.

its getting ridiculous how its so hard to find images that
will display on a science forum.

lawsuits should follow.

violations of the right to use for scientific discussion.





3/4 inch of dust build up on the moon in 4.527 billion years,LOL and QM is fantasy science.
paul #56221 07/05/16 04:36 PM
Joined: May 2011
Posts: 2,819
O
Megastar
Offline
Megastar
O
Joined: May 2011
Posts: 2,819
Compare Planks blue graph 5000K to your graph at top of atmosphere yellow (so before the light goes into the atmosphere). Your graph actually has Plancks 5250 deg C calculation on it which is that black line. There is a couple of small differences and the reason for those is even known most of which is to do with the Corona. The photons have to pass thru it.

Plancks graph

Your graph of the suns emission you used


Anyhow that is the standard science answer for what it's worth, I don't think I can help beyond that you need to do your own studying.

Bill G clearly has some other idea for the curve shape which I have never heard so you might want to get his idea to see if there is any merit. He isn't talking to me so I can't help, I am always the bad boy smile

Same as I always say I am not here to convert you so take every argument on it's merit. I don't mind what you end up believing but I have given you the true scientific view which should match up to any science article you read and hopefully they make sense now.

Last edited by Orac; 07/05/16 05:49 PM.

I believe in "Evil, Bad, Ungodly fantasy science and maths", so I am undoubtedly wrong to you.
Orac #56222 07/05/16 09:09 PM
Joined: Mar 2006
Posts: 4,136
P
paul Offline OP
Megastar
OP Offline
Megastar
P
Joined: Mar 2006
Posts: 4,136
AH HA !!!

https://en.wikipedia.org/wiki/Planck_constant

Quote:
To save his theory, Planck had to resort to using the then controversial theory of statistical mechanics,[6] which he described as "an act of despair … I was ready to sacrifice any of my previous convictions about physics.




so statistics were broken out into mainstream by Maxwell.

OK, Im out I have read enough.

you cant have a strong foundation based on statistics.







3/4 inch of dust build up on the moon in 4.527 billion years,LOL and QM is fantasy science.
paul #56223 07/06/16 01:52 AM
Joined: May 2011
Posts: 2,819
O
Megastar
Offline
Megastar
O
Joined: May 2011
Posts: 2,819
It was Max Planck not Maxwell.

Haha no problems. There is another sting in the tail you won't like anyhow which I warned you about. I suspect it will be a bigger no go for you anyhow.

On the top graph is a black line to the right marked classical theory (5000K). That is the calculation of what the emission should look like if you use classical physics thermodynamics. If you accept the result then classical thermodynamics is wrong.

Historically this is the first result science accepts in it's modern framework that produces a result that is at odds with Classical physics. That is why I and anyone who has studied high level physics knows this background intimately as it marks the start of the death of classical physics.

That is what had me shaking my head when Bill G was claiming the process was classical.

So what you need to do is come up with a theory that produces that emission curve using only classical physics and you can change the whole of science. We don't have any other way to produce that emission curve (I exclude Bill G at this point as he hasn't published his answer) other than move into the modern framework and leave classical physics behind.

Does that make my expression scary close make sense now smile

So anyhow that I believe leaves us no more common ground, and I am outta here. I will leave you to discus this now with Bill G and his theory.

Good luck with it all.

Now for some constructive feedback is there anywhere in my discussion you feel I could have simplified it further and went into too much detail?

Last edited by Orac; 07/06/16 04:14 AM.

I believe in "Evil, Bad, Ungodly fantasy science and maths", so I am undoubtedly wrong to you.
Orac #56224 07/06/16 12:22 PM
Joined: Mar 2006
Posts: 4,136
P
paul Offline OP
Megastar
OP Offline
Megastar
P
Joined: Mar 2006
Posts: 4,136



the graph shows how the atom is releasing its energy
from looking at the way that the energy is released in
the graph as the atoms cool, they slowly begin to release
tiny amounts of energy @ 0 um then there is a progressively
larger amount of energy release that begins @ apx 0.2 um
that builds to the 5000 K curve in the graph.

each of the energy releases is a separate frequency in
the spectrum.

the graph only shows the blue , green and red lines

would I be wrong if I said that each of the colored lines
in the graph represented a single energy release from the
same atom as it cools and streams energy?

would I be wrong if I said that the entire spectrum of
frequencies is a continuous stream of separate energy releases
as single atoms stream the excess energy as they cool.

so that as each atom cools it is following a precise order
of energy releases that can be considered a packet of all
frequencies.














3/4 inch of dust build up on the moon in 4.527 billion years,LOL and QM is fantasy science.
Orac #56225 07/06/16 12:41 PM
Joined: Sep 2012
Posts: 1,209
N
Megastar
Offline
Megastar
N
Joined: Sep 2012
Posts: 1,209
Originally Posted By: Orac
Originally Posted By: paul
believe it or not this has spurned an interest that I
now need to explore.

No worries his biography gives a brief outline of him.

http://www.nobelprize.org/nobel_prizes/physics/laureates/1918/planck-bio.html

You will see he was lead to the problem you are dealing with when he was studying radiation. The question of why is the suns spectrum broad and smooth.

He formalized your vibrations into a resonant element and when you do so the energy exchanges can only be in discrete packet or quanta.

Quote:
revolutionary idea that the energy emitted by a resonator could only take on discrete values or quanta. The energy for a resonator of frequency v is hv where h is a universal constant, now called Planck's constant.

So when your vibrating molecules try to unload their kinetic energy they can only do so in discrete packets and that is the origin of photons but having a broad frequency range representing the broad range of kinetic energies in the molecules.

In effect old blacksmiths already knew this because they had worked white hot metal and the composition of the metal didn't change the color it always has the same distinctive color.

This is the emission curve you get using the theory for different temperatures 5000K, 4000K, 3000K

So heat any material to 5000K and you get visible light and that explains the incandescent light bulb.

When you look at molten lava you can tell the temperature just by looking at the color you don't have to know the composition at all.

Compare your suns emission graph to the 5000K curve developed by Max from theory smile

Here lets make in simple and put on the same post:


Shame I can't get them scaled the same ... but do you see the two graphs match (your image has the 5250 deg C planck graph in black). I can tell the sun is around 5000K in temperature just from the graph. What I don't have is a clue what the sun is made of that requires a different technique and different data. Our theory says it doesn't matter what the material is the heat of 5000K will produce that curve.

So Max was the first person that could explain that emission curve of the sun and it's profound implications. Yes he was religious and we don't hold that against him. His spiritual vibrations are the foundations of QM.

Anyhow that is the scientific origin of that emission curve for the sun, which I and almost all scientists believe. I think Bill G believes something else but he can argue that.

*************************************************************

WE ARE NOT SURE ABOVE GRAPH !!!

ROTATION = DOPPLER

WE PEOPLE MAKING ROTATION IN UNIVERSE !!!
PLEASE LOOK BELOW OLD DOPPLER !!!

STUDY PLEASE BELOW PROBLEM
Distance " d " exist because sensor is making rotation CW or CCW
(below model please confirm in books - physics .
I showing here only to help You understand problem it is not new information )




REAL Hz are totaly different !!!
we are registering colours that YOU SHOWED ON ABOVE GRAPHS

BUT BUT we see this colours because we are moving in universe !!!

we making rotation ( LABORATORY where THEY READ THIS Hz is making rotation )

( notice that exist distance "d"
"+ "or "– "direction )



DOPPLER IS CHANGING REAL Hz !


Right now is 2016 and we are not sure how colour is sending atoms !
to evaluate absolute colour we must evaluate and add to physics ABSOLUTE rotation !!!



INSIDE MICHELSON MORLEY WE HAVE DOPPLER !!!
VERY GENTEL DOPPLER but not ZERO !!! I'm sure !!!

PLEASE COMPARE LEFT/RIGHT SIDE below ilustration
it is NOT ZERO !!!



EINSTEIN and LORENZ ignored ! IGNORED famous Pi !!!!!!




DOPPLER inside ATOM = FUTURE FOR PHYSICS !!!
right now I'm sure that in short time we people will start very fast
trip ( VERY VERY FAST !!!)

PLESE LOOK HOW EASY DYNAMICA IS SOLVING THREE BROTHERS PARADOXE!!!

TERMODYNAMICA inside ATOM
left mouse and right mouse is eating the same food
how works methabolism ?
how many energy atoms need to comomunicate with electrons ?
how heavy are electrons ?






WITHOUT BELOW INFORMATION WE CAN NOT SPEAK ABOUT COLOURS !!!
person inside box is able measure ALL 3D directions to look OMEGA
after he will have his OMEGA he will be sure WHAT HE SEE and WHAT COLOUR HE SEE WHEN HE LOOKING ON THE SUN !!!!





INTERNAL force !
the same problem is registering EACH ATOM

how big force is ( tension ) is registering beam ?
[img]http://1.bp.blogspot.com/-i0QAJkwDUfU/VBGtJcwO_PI/AAAAAAAAB90/GgkSTBMgMDQ/s1600/CIMG3319.JPG[/img]

ABOVE I SHOWED CLASSIC METHOD HOW TO SOLVE
FAMOUS GALILEO's PROBLEM !
WE CAN SOLVE THIS PROBLEM ALSO WE CAN USE LIGH or CLASSICA

[img]https://3.bp.blogspot.com/-u4PJ98sWLZA/V...-Q/s1600/ws.jpg[/img]


FUKO's pendulum ! MY OWN MODERNISATION
ropes are the same !!!
ropes are not moving with EARTH in space ( SIMILAR LIKE FUKO's pendulum !!!)

WE ARE ABLE READ DOPPLER !!!!

[img]https://1.bp.blogspot.com/-G2_gSCpMsa8/V...cB/s1600/33.jpg[/img]

ROPES = MATHERIAL

MATHERIAL "is DANCING" and feel dynamica !!

Air in box ( material )
is like Girl that dancing ( narow /wide arm problem)
FILM inside dark box will not register the same ENERGY
Em radiation is pusshing Air and losting Joules
( the same brightness is impossible for all 3D directions )
we can use bulb or special hydrogen portion (FLASH EXPLOSION )

[img]http://2.bp.blogspot.com/-pf0vOT4PC_4/VGmOcWetogI/AAAAAAAACII/fHj1l3ZdWd8/s1600/3.jpg[/img]

Last edited by newton; 07/06/16 01:03 PM.
newton #56226 07/06/16 01:07 PM
Joined: Sep 2012
Posts: 1,209
N
Megastar
Offline
Megastar
N
Joined: Sep 2012
Posts: 1,209



1600 Galilean relativity -----Maxwell 1861 ------- Marosz 2012


paul #56227 07/06/16 01:14 PM
Joined: Sep 2012
Posts: 1,209
N
Megastar
Offline
Megastar
N
Joined: Sep 2012
Posts: 1,209
ABOUT QUANTUM PHYSICS and PLANCK ?

YOU NO NEED LOOK ON PHOTON TO EVALUATE photon's P=mv

( when you looking on photon you changing resoults )

PLEASE MEASURE WHAT PHOTON DID !! effecte his work not
look on photon !! ( please measure effecte of his work )




physics's history...
" Galilean relativity states that the laws of motion are the same in all inertial frames. Galileo Galilei first described this principle in 1632 in his Dialogue Concerning the Two Chief World Systems using the example of a ship travelling at constant velocity, without rocking, on a smooth sea; any observer doing experiments below the deck would not be able to tell whether the ship was moving or "

1861 J. C. Maxwell, published his theory of electromagnetic fields and radiation,which shows that light has momentum and thus can exert pressure on objects.

2011 Grover Swartzlander first began to examine a revolutionary concept in optical physics after studying the flight of a moth. He watched the animal use its wings to create lift..... ( find more in google)


newton #56228 07/06/16 01:17 PM
Joined: May 2011
Posts: 2,819
O
Megastar
Offline
Megastar
O
Joined: May 2011
Posts: 2,819
Dancing around another long spam series post by Marosz.

Paul, I am having trouble with your English, it could mean several things to me so let me try and phrase what the graph shows.

The graph is the wavelength along the bottom x axis and the power/energy density on the y axis in layman terms how bright that wavelength of color is seen (assuming we are in the visible spectrum).

So lets turn this into kinetic energy (motion of your molecules) with temperature.

So at 3000K most of your molecules contain an energy which equates to energy of a photon at 1um wavelength. Some are higher some are lower but the most are at 1um, not every molecule has exactly the same temperature. So the graph shows the spectrum intensity you would expect if the sun was at 3000K in temperature.

Similarly the 5000K shows the intensity for each wavelength you would expect to measure in a sun at that temperature.

The reason for the sharp cutoff to the left is you need a lot of kinetic energy in a single molecule to release those wavelengths which are way up in the ultra violent and heading up to x-rays. You don't have any molecules with enough kinetic energy to do that at these low temperatures.

If you want to have more energy to the left you need a hotter temperature you need 7000K to get up into the mid blues.

Let me see if I can get a color reference for you ... ok got one from an LED light manufacturer which is quite good.


Basically the hotter the temperature of your sun or light element the more left on that graph your cutoff will be.

Lets add a hotter sun on the graph the blue one here is 15000K it also adds the radiation of a human body. You can do this for any body at a temperature in a lab and measure the spectrum


It all conforms to Plancks formula which is why everyone uses it in all sorts of industries.

Most engineers and all physics graduates will have done labs and learn the trick because it is widely used for contactless temperature measurement. You also need to take care if you aren't dealing with an emission but you are sending in a EM beam to create a reading off a surface as the surface emissivity has an effect and so you need graduates to know that.

Hopefully AR2 will deal with Marosz and we won't be dodging in and out of the wall of spam.

Last edited by Orac; 07/06/16 01:51 PM.

I believe in "Evil, Bad, Ungodly fantasy science and maths", so I am undoubtedly wrong to you.
paul #56229 07/06/16 01:20 PM
Joined: Sep 2012
Posts: 1,209
N
Megastar
Offline
Megastar
N
Joined: Sep 2012
Posts: 1,209
during first OLD cars moving only 20 km/h nobody ask about aerodynamica !!!

nobody feel this problem !!!

problem that we not see exist !


paul #56230 07/06/16 01:28 PM
Joined: Sep 2012
Posts: 1,209
N
Megastar
Offline
Megastar
N
Joined: Sep 2012
Posts: 1,209
Czerenkow
each body that moving is generating EM wave

Marosz
Each body that is moving is generating EM vacuum ( cavitation)


to describe cavitation please use ideal GAS model
to help You I prepared simple example
two empty balls Under Water ! ( water = EM preasure from stars )

each ball drinking energy ( inside balls we have holes ! ) water is going inside ball




IF IF EMPTY BALL WILL START VERY VERY fast rotation respect to own
axis !!! WATER ( energy will not go inside BALL )

ROTATION = photons will not able GO INSIDE BALL !!!
WATER can not WIN with F= m*V*V/ R

ROTATION = WE LOSTING MASS !!!

newton #56237 07/08/16 05:09 AM
Joined: May 2011
Posts: 2,819
O
Megastar
Offline
Megastar
O
Joined: May 2011
Posts: 2,819
Paul this took me a bit to find because incandescent globe manufacture is not high these days.

Here is the graph of an halogen incandescent globe filament running at different temperatures from a manufacturer, they will have done it by controlling the voltage to the globe.



As you can see it does a good impression of looking like the suns emission the hotter it gets smile

So there isn't much difference between the sun and the light bulb really all that has changed is the source of the heat, electricity for the light and nuclear for the sun.

The interesting part for you is there is a cycle in a halogen light very different to the sun and a normal incandescent bulb but it shows the process can have subcycles and the emission remains the same


The process simply increases the efficiency of generating heat and it's still the heat that produces the light.

Here are the emission spectrum of common lighting .. excluding LED's which will sidetrack us. The LED's looks something like the fluorescent lamp and you can see there emission looks very different to the sun and incandescent light because they aren't based on heat. If you see a broad sloped bell shape like that with a left side cutoff, it's always a thermal emission unless it's man made and trying to imitate a thermal emission.



So you could ask whatever you were trying to understand about the spectrum using an incandescent bulb if you like it's essentially the same emission process and something data will exist for.

The bottom line is unless the sun is man made its light emission is thermal smile

Last edited by Orac; 07/08/16 05:48 AM.

I believe in "Evil, Bad, Ungodly fantasy science and maths", so I am undoubtedly wrong to you.
Orac #56238 07/08/16 11:14 AM
Joined: Sep 2012
Posts: 1,209
N
Megastar
Offline
Megastar
N
Joined: Sep 2012
Posts: 1,209
SPECTRUM ???

HOW BIG FORCE IS PUSHING EARTH
( WHY GRAPH THAT YOU SHOWED NOT CONTAIN SPEED ???!!!)
speed is changing force ??
( changing colour ) changing Hz ???



Distance D = constant !
/SUN's power = constant !
/ L1 , L2 , L3 ?



SPECTRUM ??? left/right situation







Last edited by newton; 07/08/16 11:32 AM.
newton #56239 07/08/16 12:16 PM
Joined: May 2011
Posts: 2,819
O
Megastar
Offline
Megastar
O
Joined: May 2011
Posts: 2,819
AR2 can you remove the Marosz spam its getting annoying. Just random unintelligent rantings that no one can understand and we have seen 50 times.

Last edited by Orac; 07/08/16 01:36 PM.

I believe in "Evil, Bad, Ungodly fantasy science and maths", so I am undoubtedly wrong to you.
Orac #56243 07/08/16 03:27 PM
Joined: Mar 2006
Posts: 4,136
P
paul Offline OP
Megastar
OP Offline
Megastar
P
Joined: Mar 2006
Posts: 4,136
What Im saying is the below.

when an atom on the sun cools.

the cooling atom releases energy.

the energy that the atom releases is not instantaneously released
it is released over a period of time as the atom cools.

if you could view the atom in slow motion as the atom cools
then you would see the electrons of the atom changing their
orbit around the nucleus from a higher orbit to a lower orbit
and a energy release is a result.

Im not certain but I would think that the energy release
would occur first followed by the electrons reducing their
orbits.

but it may be a simultaneous balance that occurs in the
atoms excess energy as the atom cools.

the energy that is released can be matched to the
energy that would be required to cause the electrons to
move to their former higher orbit.

and the energy that is radiated away from the atom is at the
specific energy that the electrons had in their former higher orbits minus the energy that they have in their present lower orbits.

each successive energy release would be lower than the previous
energy release.

which corresponds with all of the graphs so far that
represent frequency and magnitude.

each color in the graph starts along the x line at the bottom
of the graph and it is this frequency in the graph that represent the time that the specific frequency (energy) is released from the atom.

so pretty much the entire spectrum is released from each
atom as it cools.



3/4 inch of dust build up on the moon in 4.527 billion years,LOL and QM is fantasy science.
paul #56244 07/09/16 02:53 AM
Joined: May 2011
Posts: 2,819
O
Megastar
Offline
Megastar
O
Joined: May 2011
Posts: 2,819
Ok lets say I buy that then how does the shape come about some sort of probability formula embedded in the atomic structure?

Your example above sort of assumes the orbitals are empty. The filament of an incandescent bulb Tungsten has around 74 electrons for example. You basically have 74 electrons rattling around is the same as one electron in hydrogen rattling around between levels. The problem is if you divid the heat energy by 74 into each electron based on energy the emission will be much lower than the single atom case.

What I am getting at is, an atom with one electron, is behaving the same as one with 13 or 33 or 74 or 98. It can't be just the outer electrons involved because you need all the frequencies so the larger atoms with more electrons have to push other electrons out causing multiple emissions yet the emission shape stays the same matching a formula?

I think you might want to read the formula of that graph that Planck derived about the energy relationship regardless of if you except the reason there is a ratio between the energy and the spectrum which you need to account for.

However at this forum level I am not sure it matters how you want to think of the process so long as you recognize the strange graph shape as a thermal signature it will do. So the sun emission of light is thermal and we can stop there.

The extension is are you happy the surface of the sun doing the emission is around 5000K even though we will never be able to go there and measure directly.

Last edited by Orac; 07/09/16 06:11 AM.

I believe in "Evil, Bad, Ungodly fantasy science and maths", so I am undoubtedly wrong to you.
paul #56245 07/09/16 02:20 PM
Joined: Oct 2006
Posts: 1,164
Megastar
Offline
Megastar
Joined: Oct 2006
Posts: 1,164
Originally Posted By: paul



the graph shows how the atom is releasing its energy
from looking at the way that the energy is released in
the graph as the atoms cool, they slowly begin to release
tiny amounts of energy @ 0 um then there is a progressively
larger amount of energy release that begins @ apx 0.2 um
that builds to the 5000 K curve in the graph.

each of the energy releases is a separate frequency in
the spectrum.

the graph only shows the blue , green and red lines

would I be wrong if I said that each of the colored lines
in the graph represented a single energy release from the
same atom as it cools and streams energy?

would I be wrong if I said that the entire spectrum of
frequencies is a continuous stream of separate energy releases
as single atoms stream the excess energy as they cool.

so that as each atom cools it is following a precise order
of energy releases that can be considered a packet of all
frequencies.
...it sounds like you're thinking of the unique electronic transitions, that are a characteristic of each element,
which create the emission and absorption "line spectra" such as the lower two spectra below:

...so first, yes, and then no, (but see below) on your two questions from above.
===

Paul, I think (but will defer to Orac for clarity or corrections) that your graph
is a graph representing (the spectrum from)
a lot of atoms (such as the sun)
...though I suppose you could think of it as the graph of one atom, over a lot of time, cooling from the high frequencies down to the lower frequencies. wink

The point though, should be that the graph represents the collective emissions from,
all the many atoms of, a blackbody (defined as a good absorber and emitter).

Most of the emissions are at the peak of the curve, and very few of the emissions are out at the tiny tail (or head) of the curve.

The height of the curve, directly above any particular frequency listed on the x axis,
tells you how much Intensity comes from that particular frequency.

Compared with the large number of atoms that are radiating at the frequency of the peak,
if just a few atoms are radiating (or if many atoms rarely radiate) at some other frequencies,
then the curve will be proportionally lower at those other frequencies.
===

There is more to the story, I think, about how photons are propagated ...as they convey and distribute energy about,
but this different perspective may help you better understand Orac's answers.

And, as Orac mentioned (on the difference between the theories)
here is something to consider ...in the long run:


~


Pyrolysis creates reduced carbon! ...Time for the next step in our evolutionary symbiosis with fire.
Orac #56246 07/10/16 12:34 AM
Joined: Mar 2006
Posts: 4,136
P
paul Offline OP
Megastar
OP Offline
Megastar
P
Joined: Mar 2006
Posts: 4,136
Quote:
how does the shape come about


ok, as the extremely hot atoms move closer to the surface
they are in the process of cooling , yet there is still
a way to go until they reach the surface so a portion of
the energy or heat that the atoms release is transfered into
the cooler atoms above them.

this continues as the atoms stream energy while being
transported to the surface very fast.

this causes the curve that you see in the graph.

perhaps.

think about holding a water hose under water and pointing
the water stream upwards , now slowly move the hose upwards
you will see the stream of water lengthens as the nozzle
gets closer to the surface of the water.

kind of like that anyway.

I think that a hydrogen atom would work just fine
it could store energy really well simply by increasing
its size ( electron orbit ).

and I dont think that there is any absorption or emission
occurring so it can only be thermal energy that is released.

the more I think about it the more I like the idea of single
atoms streaming the full spectrum.

and what I mean above is that a single atom streams the
entire spectrum of frequencies as it cools.

like this...

a1 streams the entire spectrum
followed by
a2 streams the entire spectrum
followed by
a3 streams the entire spectrum
followed by
a4 streams the entire spectrum
followed by
a5 streams the entire spectrum
etc...

so basically the highest intensity energy release is the
first to go then each successive energy release is lower
that the previous energy release.

this way we would get tightly packaged packets
of frequencies ( vibrations ) and these are
what we call photons.

ok ,think about the atoms being strobe lights that
are powered by a battery that is quickly consumed
... each flash of light being dimmer than the previous
flash of light.

only the atom is releasing its energy as vibrations.

that we call thermal energy.





3/4 inch of dust build up on the moon in 4.527 billion years,LOL and QM is fantasy science.
paul #56247 07/10/16 02:36 AM
Joined: May 2011
Posts: 2,819
O
Megastar
Offline
Megastar
O
Joined: May 2011
Posts: 2,819
For a layman that is close enough and you have got it that the process is atomic/molecule level.

It has to be because in classical physics when you measure temperature (heat) with a thermometer etc it is translated as the kinetic energy of movement speed of the molecules/atoms in the gas/solid/liquid not the speed of the electrons around the orbitals (energy levels of the electrons) in the atom/molecule.

In classical physics the explaination involved with thermal radiation is that if you take a gas of atoms/molecules and confine it to a region of space (solids and liquids do that automatically) containing some radiation field with some characteristic temperature, the atoms and the radiation will eventually come to some equilibrium in which the kinetic energy distribution of the atoms and the frequency spectrum of the radiation will have the same characteristic temperature.

In that explaination lies the fact the radiation and the kinetic energy are both forms of energy and are actively connected.

The problem for classical physics is there exist no formula and no explaination for the connection and hence no reason for the shape of the spectrum. If you assume they are connected your mathematics will fail as that shape is very strange it has some very strange properties.

What Planck did was work out that if one makes a premise of a resonant vibration that has distinct steps (quantization) the mathematics gives you that shape of emission. So his logic was there must be a resonant connection between the molecule/atom kinetic energy and the thermal spectrum (AKA light) and that set the stage to then try and measure and quantify that connection, what it is and how it works.

Planck had removed a foundation stone of classical physics and it largely went unnoticed for another twenty-five years until classical physics would ultimately collapse as it would be connected to other inconsistencies that were found.

Last edited by Orac; 07/10/16 03:14 AM.

I believe in "Evil, Bad, Ungodly fantasy science and maths", so I am undoubtedly wrong to you.
Orac #56248 07/10/16 08:05 PM
Joined: Mar 2006
Posts: 4,136
P
paul Offline OP
Megastar
OP Offline
Megastar
P
Joined: Mar 2006
Posts: 4,136
Quote:
What Planck did was work out that if one makes a premise of a resonant vibration that has distinct steps


carry that a little further and you can have distinct lines
that can be translated into distinct frequencies / vibrations
as in hyperfine lines of electron orbits.

we should recognize that as the electron drops to a lower
orbit it would not follow a precisely calculated change
in energy level ... it simply changes its energy level
as the atom cools and releases thermal energy ( vibrations ).

so instead of the electron dropping to a energy level
that corresponds with known observations over a period
of time , the electron is dropping to that known energy level
but not all at one time.

the electron is going through many energy level changes
in order to reach that known energy level in that period of time.

and each time the electron drops slightly there is a
corresponding energy release of thermal energy ( vibrations)
that are each lower than the previous energy release.

and the energy level changes are due to the cooling of
the atom.


and this is how the full spectrum is emitted from the sun
and enters the earths atmosphere.

and this is why co2 absorbs inbound radiation from the sun.

causing the earth to cool.

by blocking the thermal heat from the sun.

the more co2 there is in the atmosphere the cooler
the earth will become.

sam wont like this ... oh well.

neither will the tens of thousands of people who depend
on the climate change due to co2 = warming + scam money
funding opportunities that supply them with a income.

oh well.

thats where I was going with this.


3/4 inch of dust build up on the moon in 4.527 billion years,LOL and QM is fantasy science.
paul #56249 07/11/16 02:41 AM
Joined: May 2011
Posts: 2,819
O
Megastar
Offline
Megastar
O
Joined: May 2011
Posts: 2,819
I don't do politics and conspiracy theories ... I will leave that to someone who may be interested to discuss.


I believe in "Evil, Bad, Ungodly fantasy science and maths", so I am undoubtedly wrong to you.
Orac #56250 07/11/16 03:34 AM
Joined: Mar 2006
Posts: 4,136
P
paul Offline OP
Megastar
OP Offline
Megastar
P
Joined: Mar 2006
Posts: 4,136
it would be nice if science wasnt involved in politics
or conspiracy theories but the facts are the facts and
when the funding money is mostly granted to those who
promote climate change and warming due to co2 and when
politics also is geared to accept the findings that co2
is causing warming and mostly demanding those findings
by mostly funding the applications that are seeking to study
the effects of co2 as a reason for the warming
(that isnt happening) then it sets up the conspiracy through politics.

the government funding should not be biased and should be
seeking actual effects not wanted effects.

the fact that we pretty much hashed this out in a few days
sort of tells the whole story about the deception within
the climate change due to co2 causing warming cartel.


3/4 inch of dust build up on the moon in 4.527 billion years,LOL and QM is fantasy science.
paul #56251 07/12/16 03:21 PM
Joined: Oct 2006
Posts: 1,164
Megastar
Offline
Megastar
Joined: Oct 2006
Posts: 1,164
Originally Posted By: paul
"What Planck did was work out that if one makes a premise of a resonant
vibration that has distinct steps" -Orac


carry that a little further and you can have distinct lines
that can be translated into distinct frequencies / vibrations
as in hyperfine lines of electron orbits.

we should recognize that as the electron drops to a lower
orbit it would not follow a precisely calculated change
in energy level ... it simply changes its energy level
as the atom cools and releases thermal energy ( vibrations ).

so instead of the electron dropping to a energy level
that corresponds with known observations over a period
of time , the electron is dropping to that known energy level
but not all at one time.

the electron is going through many energy level changes
in order to reach that known energy level in that period of time.

and each time the electron drops slightly there is a
corresponding energy release of thermal energy ( vibrations)
that are each lower than the previous energy release.

and the energy level changes are due to the cooling of
the atom.

and this is how the full spectrum is emitted from the sun
and enters the earths atmosphere.

and this is why co2 absorbs inbound radiation from the sun.

causing the earth to cool.

by blocking the thermal heat from the sun.

the more co2 there is in the atmosphere the cooler
the earth will become.

sam wont like this ... oh well.

neither will the tens of thousands of people who depend
on the climate change due to co2 = warming + scam money
funding opportunities that supply them with a income.

oh well.

thats where I was going with this.
...methinks thou doth protest so much, Paul, that your agenda is obvious!

Your efforts to deny or overlook the validity of the sciences in general,
and to also contrive your own "just so" stories of science, must have a purpose; shocked
because your stories seem to always support your agenda—wherever you are "going with this."

===

Paul, it is nice you can see, in whatever way, that an “atom is releasing its energy” in a process we observe
as thermal energy, or “we call thermal energy,” in the form of electromagnetic radiation we call photons.

Perhaps it is just easier for you to think of one atom, cooling progressively, but I think that logic doesn’t work.
If an atom emitted that “highest energy” photon to begin with, it would then become a cooler, low energy atom.
Where would the energy to emit the next “almost highest energy,” or “next-highest energy,” photon come from?

The picture should work better, if you see how, for any large group of atoms (at some average temperature),
at any given time the group will consist of some atoms that are hotter than average and others that are cooler than average.

Those hotter and cooler atoms radiate at frequencies that create the head and tail of the curve/spectrum,
while most of the atoms (the peak intensity for the curve) will be radiating at (or nearer to) the frequency
for that average temperature.
===

But it's good you seem to better understand thermal radiation now, and whatever story you need to tell yourself
(however inaccurate it may be), at least we can agree on the nature of the spectrum it generates.

Though Paul, even if we accept your ideas about "hyperfine lines" your logic still doesn't follow.
How does your explanation, about atoms radiating heat, suddenly relate to the topic of CO2?
And what does your point about how "co2 absorbs inbound radiation from the sun" have to do with anything
that I might not like about greenhouse warming?

The greenhouse effect is driven by the "upgoing" long-wave heat,
but not by the "downgoing" short-wave heat. What is your point?



We've been over this before, several times, but maybe this new picture will help
you see why the incoming heat, in sunlight, isn't relevant as you had suggested above.
~ wink

===
And thank you Orac, for your most interesting and informative post:
Originally Posted By: Orac
For a layman that is close enough and you have got it that the process is atomic/molecule level.

It has to be because in classical physics when you measure temperature (heat) with a thermometer etc it is translated as the kinetic energy of movement speed of the molecules/atoms in the gas/solid/liquid not the speed of the electrons around the orbitals (energy levels of the electrons) in the atom/molecule.

In classical physics the explaination involved with thermal radiation is that if you take a gas of atoms/molecules and confine it to a region of space (solids and liquids do that automatically) containing some radiation field with some characteristic temperature, the atoms and the radiation will eventually come to some equilibrium in which the kinetic energy distribution of the atoms and the frequency spectrum of the radiation will have the same characteristic temperature.

In that explaination lies the fact the radiation and the kinetic energy are both forms of energy and are actively connected.

The problem for classical physics is there exist no formula and no explaination for the connection and hence no reason for the shape of the spectrum. If you assume they are connected your mathematics will fail as that shape is very strange it has some very strange properties.

What Planck did was work out that if one makes a premise of a resonant vibration that has distinct steps (quantization) the mathematics gives you that shape of emission. So his logic was there must be a resonant connection between the molecule/atom kinetic energy and the thermal spectrum (AKA light) and that set the stage to then try and measure and quantify that connection, what it is and how it works.

Planck had removed a foundation stone of classical physics and it largely went unnoticed for another twenty-five years until classical physics would ultimately collapse as it would be connected to other inconsistencies that were found.
good, on topic, sunlight ...certainly worthy of repeating.
~ cool


Pyrolysis creates reduced carbon! ...Time for the next step in our evolutionary symbiosis with fire.
samwik #56252 07/13/16 03:04 AM
Joined: Mar 2006
Posts: 4,136
P
paul Offline OP
Megastar
OP Offline
Megastar
P
Joined: Mar 2006
Posts: 4,136
Quote:
Where would the energy to emit the next “almost highest energy,” or “next-highest energy,” photon come from?


the same place !!!

the atom continues to release its excess energy.

Quote:
Those hotter and cooler atoms radiate at frequencies that create the head and tail of the curve/spectrum,
while most of the atoms (the peak intensity for the curve) will be radiating at (or nearer to) the frequency
for that average temperature.


what would be the difference between examining one atom as it
cools and examining a group of atoms as they cool.

I would think that the one would represent the group.

by examining a group of atoms all your going to do is
make the examination more difficult and harder to understand.

not that you would intentionally do anything like that.

Quote:
Though Paul, even if we accept your ideas about "hyperfine lines" your logic still doesn't follow.


its not mine.

https://en.wikipedia.org/wiki/Hyperfine_structure

Quote:
In atomic physics, hyperfine structure is the different effects leading to small shifts and splittings in the energy levels of atoms, molecules and ions.


the atoms we are discussing were not super heated by photon
absorption that caused the electron to increase its orbit
they were slowly heated after they cooled enough to hold
an electron.

and photon emission from the atom is due to a continuous
cooling.

the atoms in its proximity are also cooling from a
super heated / energized state and most likely cannot absorb
a photon so the photon is radiated away from the atom/atoms.

Quote:
And what does your point about how "co2 absorbs inbound radiation from the sun" have to do with anything
that I might not like about greenhouse warming?


I was just thinking that you wouldnt like it because it
isnt in line with your beliefs because the inbound
radiation causes the co2 in the atmosphere
to become saturated as they become energized and that
isnt what is being preached by the global warming cartel .

so all day long ( daylight ) the normal state (non energized)
co2 molecules in the atmosphere will be blocking the inbound photons from the sun that they absorb.

and the energized co2 in the atmosphere cannot
absorb another photon until they emit a photon.

but as soon as co2 in the atmosphere emits
a photon in the daytime it is re-energized by inbound
radiation from the sun.

Quote:
The greenhouse effect is driven by the "upgoing" long-wave heat,
but not by the "downgoing" short-wave heat. What is your point?


my point is that this pretty much removes your entire greenhouse effect during daylight hours.

but if its any consolation to you and yours the co2 will
warm the planet at night if it emits a photon towards the
earth.

but all day long the co2 pretty much just blocks the radiation
from the sun.

so in order to debunk this the global warming due to co2 cartel
will need to show that the sun does not emit the full spectrum
and that it only emits photons from individual
atoms in all the required modes to produce all the required frequencies from the available atoms that actually are on the
sun that just so happen to only have the frequencies that would allow the greenhouse effect to work the way that they believe
or want it to work.

because if the sun emits the full spectrum then the co2 in
the atmosphere will absorb the emitted photons that are in
the required frequency.



3/4 inch of dust build up on the moon in 4.527 billion years,LOL and QM is fantasy science.
paul #56254 07/13/16 09:23 AM
Joined: Oct 2006
Posts: 1,164
Megastar
Offline
Megastar
Joined: Oct 2006
Posts: 1,164
Originally Posted By: paul
Quote:
Where would the energy to emit the next “almost highest energy,” or “next-highest energy,” photon come from?


the same place !!!

the atom continues to release its excess energy.
What place? Is your single atom isolated and cooling only, or is it being constantly supplied with excess energy?
Some of your deductions make sense from only one perspective,
but then different conclusions you post only make sense from the other perspective.


Originally Posted By: paul
Quote:
Those hotter and cooler atoms radiate at frequencies that create the head and tail of the curve/spectrum,
while most of the atoms (the peak intensity for the curve) will be radiating at (or nearer to) the frequency
for that average temperature.


what would be the difference between examining one atom as it
cools and examining a group of atoms as they cool.

I would think that the one would represent the group.
Because the “group of atoms” isn’t cooling, is it?
Is the sun cooling? I think over the past few billion years the sun has been increasing its temperature.
If I recall correctly, paleoclimatology shows that the “solar constant” is still steadily increasing.

So how do you get that continuous spectrum, covering a broad range of frequencies (and temperatures),
if you must wait for each single atom to cool enough to have lost all the energy that we see in every snapshot of the solar spectrum.


Originally Posted By: paul
by examining a group of atoms all your going to do is
make the examination more difficult and harder to understand.

not that you would intentionally do anything like that.
Well....
It is the difference between examining reality,
or examining your speculations, based on oversimplified thought experiments, on how you see reality operating.


Originally Posted By: paul
Quote:
Though Paul, even if we accept your ideas about "hyperfine lines" your logic still doesn't follow.


its not mine.

https://en.wikipedia.org/wiki/Hyperfine_structure

Quote:
In atomic physics, hyperfine structure is the different effects leading to small shifts and splittings in the energy levels of atoms, molecules and ions.
Congratulations Paul!
It’s good to see you’re doing some real research.
That sounds like good information, and it seems like that might help explain a lot.

But do you think every atom goes through every possible combination of hyperfine divisions, as it cools, or just the isolated atom you’d be measuring?
I don’t think you can get the full solar spectrum from just one atom, even if you wait long enough for it to cool all the way to absolute zero.


Originally Posted By: paul
the atoms we are discussing were not super heated by photon
absorption that caused the electron to increase its orbit
they were slowly heated after they cooled enough to hold
an electron.

and photon emission from the atom is due to a continuous
cooling.

the atoms in its proximity are also cooling from a
super heated / energized state and most likely cannot absorb
a photon so the photon is radiated away from the atom/atoms.
Well, I’m glad to see you’re maybe starting to see the need for a group of atoms,
if you want to describe what is really creating the solar spectrum.


Originally Posted By: paul
Quote:
And what does your point about how "co2 absorbs inbound radiation from the sun" have to do with anything
that I might not like about greenhouse warming?

I was just thinking that you wouldnt like it because it
isnt in line with your beliefs because the inbound
radiation causes the co2 in the atmosphere
to become saturated as they become energized and that
isnt what is being preached by the global warming cartel .

so all day long ( daylight ) the normal state (non energized)
co2 molecules in the atmosphere will be blocking the inbound photons from the sun that they absorb.

and the energized co2 in the atmosphere cannot
absorb another photon until they emit a photon.

but as soon as co2 in the atmosphere emits
a photon in the daytime it is re-energized by inbound
radiation from the sun.

Quote:
The greenhouse effect is driven by the "upgoing" long-wave heat,
but not by the "downgoing" short-wave heat. What is your point?

my point is that this pretty much removes your entire greenhouse effect during daylight hours.

but if its any consolation to you and yours the co2 will
warm the planet at night if it emits a photon towards the
earth.

but all day long the co2 pretty much just blocks the radiation
from the sun.

so in order to debunk this the global warming due to co2 cartel
will need to show that the sun does not emit the full spectrum
and that it only emits photons from individual
atoms in all the required modes to produce all the required frequencies from the available atoms that actually are on the
sun that just so happen to only have the frequencies that would allow the greenhouse effect to work the way that they believe
or want it to work.
Well, that is a relief!
So I don’t need to worry,
since your latest “just so” story is as flawed as most of your “interpretations” of how reality works.

By your logic here, a greenhouse shouldn’t work either, since it is blocking all that heat coming from the sun. wink


Originally Posted By: paul
because if the sun emits the full spectrum then the co2 in
the atmosphere will absorb the emitted photons that are in
the required frequency.
I know that speaking theoretically about blackbody radiation,
we’ve talked about how “the full spectrum” is emitted, at least perhaps at some minimal intensity.


But Paul, did you look at that spectrum of the sun you posted?
Do you see what those pictures posted on this thread show?

The only solar energy that is blocked by CO2 is shorter than (to the left of) 3 microns.
And there is very little “Intensity,” coming from the sun, at those wavelengths where CO2 would block any incoming part of the spectrum.

The planet cools by radiating away the longer-wave heat (to the right of 3 microns) from the planet, as the graph I posted above shows.

It is this longer-wave heat loss that is relevant to the greenhouse effect, as the graph above shows,
since that outgoing heat is being blocked more and more by higher CO2 levels.

Do you not see the huge difference, on that graph above, between the left (incoming) and right (outgoing) sides?
~

p.s. It is good to hear you admit that "the co2 will warm the planet at night," at least.
Even if the warming occurred for just an hour, it apparently is enough to be warming the oceans and the crust,
as well as warming the air and melting our planet's ice reserves, on average, 24/7/365 globally.

Last edited by samwik; 07/13/16 10:31 AM. Reason: add p.s.

Pyrolysis creates reduced carbon! ...Time for the next step in our evolutionary symbiosis with fire.
samwik #56255 07/13/16 02:36 PM
Joined: Mar 2006
Posts: 4,136
P
paul Offline OP
Megastar
OP Offline
Megastar
P
Joined: Mar 2006
Posts: 4,136
Sam ,I do wish that you would stop taking things out of context
because the readers probably still have the ability to comprehend
the meanings of a reply.

Originally Posted By: Sam
Where would the energy to emit the next “almost highest energy,” or “next-highest energy,” photon come from?


my reply to your question above was...

Originally Posted By: paul
the same place !!!

the atom continues to release its excess energy.


I believe that my reply was clear enough as we already know
that the atom releases its excess energy as it cools.

Originally Posted By: Sam
Because the “group of atoms” isn’t cooling, is it?


yes the group of atoms are cooling but when you use the words
"group of atoms" I think that would be sort of misleading
and the words
(layers of atoms at certain temperature ranges) should be used
instead.

you certainly must not think that it would be likely that
any two atoms located within a distance of a mile or so
would have a temperature differential that would be significant.

I certainly dont.

Quote:
Well, I’m glad to see you’re maybe starting to see the need for a group of atoms,
if you want to describe what is really creating the solar spectrum.


not really as the temperature differences that would be felt by a "group of atoms" would not be local.
.

the temperature differences would be in layers where the
temperatures gradually decrease as you get closer to the suns surface.

Quote:
By your logic here, a greenhouse shouldn’t work either, since it is blocking all that heat coming from the sun.


no, a greenhouse does not block the heat from the sun
it is an enclosure that holds the heat from the
sun inside the enclosure.

note: glass is not used in the below example.

if you fill that enclosure with nothing but co2 molecules
then the co2 molecules will absorb the frequencies
that a co2 molecule can absorb and at the same time it becomes
a sun blocker because it has prevented that amount of the suns
heat from reaching the inside surfaces of the greenhouse.

the important part in the above is that the excited co2
molecules cannot absorb another photon until they emit a
photon ...

once the co2 molecules have become saturated they cannot
absorb more sunlight so portions of the sunlight will reach
the inside surfaces of the greenhouse causing the inside
surfaces of the greenhouse to warm slightly.

and when the inside surfaces of the greenhouse do emit
back radiation the co2 cannot absorb that back radiation and
the back radiation exits the greenhouse.

Quote:
The planet cools by radiating away the longer-wave heat (to the right of 3 microns) from the planet, as the graph I posted above shows.


from what I understand you seem to be saying that the full
spectrum that the sun emits does not include the long wave
frequencies that the co2 in the atmosphere could absorb.

but it is my understanding that sunlight does include all
of the frequencies in the spectrum and even though there
is a tiny amount of co2 in the atmosphere the suns light
is everywhere in the atmosphere and all of the co2 becomes
excited / saturated by the incoming sun light so the back
radiation will not be absorbed by the already excited
co2 molecules in the atmosphere and the amount of emitted
heat from the earths surface that could be absorbed by the
co2 molecules in the atmosphere if the co2 molecules were not
already saturated leaves the earth.


co2 causes cooling not warming.

so co2 basically holds heat during the day and at night
it becomes able to radiate that stored energy to the
earths surface where the energy warms the earth and at night
the co2 can re-absorb re-emit the emitted heat from the earths
surface over and over again , it just cant do that on the
sun lit side of the earth.




3/4 inch of dust build up on the moon in 4.527 billion years,LOL and QM is fantasy science.
paul #56256 07/13/16 07:12 PM
Joined: Dec 2010
Posts: 1,858
B
Megastar
Offline
Megastar
B
Joined: Dec 2010
Posts: 1,858
Originally Posted By: paul
Sam ,I do wish that you would stop taking things out of context
because the readers probably still have the ability to comprehend
the meanings of a reply.

Originally Posted By: Sam
Where would the energy to emit the next “almost highest energy,” or “next-highest energy,” photon come from?


my reply to your question above was...

Originally Posted By: paul
the same place !!!

the atom continues to release its excess energy.



One problem. When an atom emits the highest possible energy photon it has to drop to its lowest energy state to do so. Therefore there is no energy available to emit any more photons until it has been re-energized.

Bill Gill


C is not the speed of light in a vacuum.
C is the universal speed limit.
Bill #56257 07/13/16 11:38 PM
Joined: Mar 2006
Posts: 4,136
P
paul Offline OP
Megastar
OP Offline
Megastar
P
Joined: Mar 2006
Posts: 4,136
Quote:
One problem. When an atom emits the highest possible energy photon it has to drop to its lowest energy state to do so.


that wouldnt happen because the atom cools as it gets closer to
the surface of the sun not all of a sudden and the only reason
that the atom does emit photons is because it is cooling and
releasing its excess energy.

so it would be a gradual process of many photon emissions
as the atom cools not a single photon emission.

the emissions would occur rapidly however but the atom
would emit the full spectrum as it cools.

so the photon emission is bound by and controlled by the temperature gradient
that the atom encounters as it gets closer to the suns surface.



3/4 inch of dust build up on the moon in 4.527 billion years,LOL and QM is fantasy science.
paul #56259 07/15/16 03:01 AM
Joined: May 2011
Posts: 2,819
O
Megastar
Offline
Megastar
O
Joined: May 2011
Posts: 2,819
Now all layman need to come up with is a way to connect temperature to energy of orbitals laugh

1.) Ionization energy is qualitatively defined as the amount of energy required to remove a bound electron from it's stable level

2.) Temperature is an objective comparative measure of the kinetic energy of the particles in a substance. It is the movement of the WHOLE molecule.

Ionization does not equal temperature except apparently to layman smile

Lets give you an image of temperature to have in your head, and when that molecule bangs into the fluid in a thermometer it makes the column of liquid expand because they are jiggling more. That is how you layman most often measure temperature.



The emission from the sun is caused by TEMPERATURE and the emission is EXACTLY the same as an incandescent bulb. If you decrease the temperature of the bulb filament or the sun they emit differently.

So using the incandescent light bulb explain how the TEMPERATURE becomes LIGHT for me please ... this I have to hear.

I will let you in a big secret there are multiple ways to emit light that don't involve electron orbitals. Try looking at Sonoluminescence, Triboluminescence and Piezoluminescence which are all forms of mechanical release of light. There are also a number of electrical, magnetic, nuclear and chemical that don't involve electron orbitals at all. In fact it is fair to say atomic orbitals are only involved in a handfull of the hundreds of way you can cause the emission of EM and it is in the minority. You use radio, tv and a microwave probably everyday and none of those involve atomic orbitals either, yet all have emission of EM.

THE MEMO IS .... YOU DON'T NEED ATOMIC ORBITALS TO GENERATE LIGHT (ELECTROMAGNETIC WAVES)

So my question is why do you keep getting the atomic orbitals involved .. evidence please using the incandescent bulb?

This was my problem with Bill G's original answer the moment he used the word orbitals IT IS NOT THE STANDARD SCIENCE ANSWER, you don't need orbitals. A particle (which has no electrons or orbitals) behaves exactly the same way and gives off the same tell tale thermal emission in DEFIANCE OF THE ANSWER ABOVE WHICH SAYS ORBITALS ARE INVOLVED.

The scientific simple answer is when any charged particle moves it creates electromagnetic radiation as it is changing the EM fields, all matter contains charged particles ... so temperature like all motion causes an emission, DOH smile

Yes it really is that simple but what that answer doesn't tell you is why it is quantized and at those frequencies patterns which Mr Planck solved.

You may now be able to understand a white dwarf which has no electrons left and no nuclear reaction yet it still glows bright white ..... wow who would predict that ... the orbitals the orbitals master they are gone.
https://en.wikipedia.org/wiki/White_dwarf

So science says any answer that involves the word ORBITAL is wrong and we have a pile of formal proofs.

Disclaimer: No atomic orbitals were required or hurt in the making of this post.

Last edited by Orac; 07/15/16 07:07 AM.

I believe in "Evil, Bad, Ungodly fantasy science and maths", so I am undoubtedly wrong to you.
Orac #56261 07/15/16 01:03 PM
Joined: May 2011
Posts: 2,819
O
Megastar
Offline
Megastar
O
Joined: May 2011
Posts: 2,819
Paul I should say your last answer doesn't involve orbitals so it is at least better than the responses from Sam and Bill G. Why do the dam orbitals keep coming up people, give it up.

It's not hard it's pretty much explained in one paragraph in the thermal radiation page lets highlight the key bits
https://en.wikipedia.org/wiki/Thermal_radiation
Quote:
Thermal radiation is the emission of electromagnetic waves from all matter that has a temperature greater than absolute zero.[3] It represents a conversion of thermal energy into electromagnetic energy. Thermal energy consists of the kinetic energy of random movements of atoms and molecules in matter. All matter with a temperature by definition is composed of particles which have kinetic energy, and which interact with each other. These atoms and molecules are composed of charged particles, i.e., protons and electrons, and kinetic interactions among matter particles result in charge-acceleration and dipole-oscillation. This results in the electrodynamic generation of coupled electric and magnetic fields, resulting in the emission of photons, radiating energy away from the body through its surface boundary. Electromagnetic radiation, including light, does not require the presence of matter to propagate and travels in the vacuum of space infinitely far if unobstructed.


It's pretty dam simple even a layman should be able to understand it yet somehow the layman keep bringing us back to the stupid atomic orbitals.

Single charged particles clumped together will give off thermal emission and they don't even have any orbitals ... it has nothing to do with orbitals.

CHARGE PARTICLE + MOVEMENT is the answer for goodness sake it's not rocket science people. I have never seen layman make such a simple thing so hard, have none of you ever moved a magnet near a wire at school and measured what happens?

If I see one more post about thermal emission that includes the word orbital or electrons, I think I will request AR2 to ban the person for gross stupidity.

The thermal emission is because lots of charged particles are jiggling around being accelerated and decelerated and because of that they emit EM. You make them hot they jiggle more and emit more .... NOT HARD PEOPLE.

Funny conclusion to that if you are smart is waving your hand gives off EM smile
Someone even got it right here ... something that would never happen on this forum.
http://physics.stackexchange.com/questions/200217/can-i-produce-radio-waves-by-waving-my-hand

So the take home lesson is there are hundreds of ways to produce EM and very few involve orbitals.

END OF STORY.

Last edited by Orac; 07/15/16 01:30 PM.

I believe in "Evil, Bad, Ungodly fantasy science and maths", so I am undoubtedly wrong to you.
Orac #56262 07/15/16 01:30 PM
Joined: Mar 2006
Posts: 4,136
P
paul Offline OP
Megastar
OP Offline
Megastar
P
Joined: Mar 2006
Posts: 4,136
Quote:
Now all layman need to come up with is a way to connect temperature to energy of orbitals


why should that be the responsibility of laymen ?


Quote:
1.) Ionization energy is qualitatively defined as the amount of energy required to remove a bound electron from it's stable level


in this thread there are no atoms in "it's stable level"


Quote:
2.) Temperature is an objective comparative measure of the kinetic energy of the particles in a substance. It is the movement of the WHOLE molecule.


in this thread I dont think there are any molecules when the focus is at or near the surface of the sun.

Quote:
Lets give you an image of temperature to have in your head, and when that molecule bangs into the fluid in a thermometer it makes the column of liquid expand because they are jiggling more. That is how you layman most often measure temperature.


the liquid expands due to a change in energy level of the atoms in the molecules inside the thermometer.
the liquid wouldnt expand simply because the molecules are jiggling around.

Quote:
The emission from the sun is caused by TEMPERATURE and the emission is EXACTLY the same as an incandescent bulb. If you decrease the temperature of the bulb filament or the sun they emit differently.


I wouldnt say exactly the same , the filament of a incandescent bulb is heated up because of the electrical current that passes
through the thin filament.

Quote:
So using the incandescent light bulb explain how the TEMPERATURE becomes LIGHT for me please ... this I have to hear.


a force is applied with a angular motion to the shaft of a DC generator.
the generator "generates" pulses of electric current due to the lines of magnetic force cutting
through the fields of a coil of copper wire inside the generator.

it is not a continuous current it breaks at a frequency that is controlled by the rpm's of the generator.

the incandescent bulb is then fed this pulsing current that passes through the filament of the bulb.
the bulb heats up and cools repeatedly over and over as the current pulses through the filament.

the electrons of the atoms in the filament pass the charge of the pulsing current from one atom
to the next atom.

this causes the electrons of the atoms to build excessive energy while the current is pulsing
current in sync with the generator and when the pulse has stopped the atoms release the excessive
energy that has built up.

this occurs over and over like it does on the sun.

think of this ball as being a filament being heated up , notice it does not glow white hot
until the heat source is removed.




Quote:
You may now be able to understand a white dwarf which has no electrons left and no nuclear reaction yet it still glows bright white ..... wow who would predict that ... the orbitals the orbitals master they are gone.


in the above video the ball only glows white hot after the heat source has been removed
are they sure that the dwarf has no electrons? couldnt the dwarf simply be in its final cooling stage like the ball.

Quote:
So science says any answer that involves the word ORBITAL is wrong and we have a pile of formal proofs.


science says a lot of things and science uses the word "ORBIT" and "ORBITS" and "ORBITAL" all the time and science
teaches the same ... so science needs to correct itself before science starts telling its students that THEY are wrong.








3/4 inch of dust build up on the moon in 4.527 billion years,LOL and QM is fantasy science.
paul #56263 07/15/16 01:32 PM
Joined: May 2011
Posts: 2,819
O
Megastar
Offline
Megastar
O
Joined: May 2011
Posts: 2,819
Originally Posted By: paul
science says a lot of things and science uses the word "ORBIT" and "ORBITS" and "ORBITAL" all the time and science
teaches the same ... so science needs to correct itself before science starts telling its students that THEY are wrong.

So how do you get a heat emission OFF something that doesn't have an orbital and I can give you thousands of those situations?

Second show me any science literature that says thermal emission has anything to do with orbitals. The references above certainly don't and all the comments on this stupid forum are about almost nothing but orbitals. Even Bill G wants to talk about orbitals and levels and he is supposedly a man who knows his science.

THE ANSWER is CHARGE + MOTION .. that is all you need to know ... seriously people.

Anyhow this has gone beyond stupid, I will leave you to it. I actually can't tolerate these levels of layman disease.

I think what this forum shows is just how set in their ways people on this forum are even when you can easily show that are blatantly and clearly wrong. The first claim by everyone is I am not wrong, even if they have to resort to ignoring the blinding obvious fact they are DEAD WRONG.

So talking about orbitals with heat emission is wrong ... I don't care how many way any idiot wants to get them involved they aren't.

Last edited by Orac; 07/15/16 01:46 PM.

I believe in "Evil, Bad, Ungodly fantasy science and maths", so I am undoubtedly wrong to you.
Orac #56264 07/15/16 01:43 PM
Joined: Mar 2006
Posts: 4,136
P
paul Offline OP
Megastar
OP Offline
Megastar
P
Joined: Mar 2006
Posts: 4,136
Quote:
Second show me any science literature that says thermal emission has anything to do with orbitals.


how about the emission of a photon by any atom!

when a hydrogen atom changes its energy level its
electron changes its orbit.

do you need me to post an article?

basically the orbit change IS THE CHANGE IN ENERGY LEVEL !!!!

the emitted photon is the result of the energy level change.


3/4 inch of dust build up on the moon in 4.527 billion years,LOL and QM is fantasy science.
paul #56265 07/15/16 01:47 PM
Joined: May 2011
Posts: 2,819
O
Megastar
Offline
Megastar
O
Joined: May 2011
Posts: 2,819
That is ionisation NOTHING TO DO WITH HEAT EMISSION.

It produces a single frequency .. MY ORIGINAL COMPLAINT to Bill G.

Stop mixing up different things which was also my complaint at the outset.

Last edited by Orac; 07/15/16 01:48 PM.

I believe in "Evil, Bad, Ungodly fantasy science and maths", so I am undoubtedly wrong to you.
Orac #56266 07/15/16 01:48 PM
Joined: Mar 2006
Posts: 4,136
P
paul Offline OP
Megastar
OP Offline
Megastar
P
Joined: Mar 2006
Posts: 4,136
Quote:
THE ANSWER is CHARGE + MOTION .. that is all you need to know ... seriously people.


oh thank you , now all I ever need to do is remember that
charge and motion is the answer to everything.

brilliant!


3/4 inch of dust build up on the moon in 4.527 billion years,LOL and QM is fantasy science.
paul #56267 07/15/16 01:50 PM
Joined: May 2011
Posts: 2,819
O
Megastar
Offline
Megastar
O
Joined: May 2011
Posts: 2,819
Originally Posted By: paul
oh thank you , now all I ever need to do is remember that charge and motion is the answer to everything.

brilliant!

Well it's better than GOD.

Now you are just trolling and so we have probably reach the I don't give a rats what you believe point and exit.


I believe in "Evil, Bad, Ungodly fantasy science and maths", so I am undoubtedly wrong to you.
Orac #56268 07/15/16 01:51 PM
Joined: Mar 2006
Posts: 4,136
P
paul Offline OP
Megastar
OP Offline
Megastar
P
Joined: Mar 2006
Posts: 4,136
Quote:
That is ionisation NOTHING TO DO WITH HEAT EMISSION.

It produces a single frequency .. MY ORIGINAL COMPLAINT to Bill G.


thanks again.

and as the atom cools the atom produces all of the frequencies
in the spectrum.



3/4 inch of dust build up on the moon in 4.527 billion years,LOL and QM is fantasy science.
Orac #56269 07/15/16 01:55 PM
Joined: Mar 2006
Posts: 4,136
P
paul Offline OP
Megastar
OP Offline
Megastar
P
Joined: Mar 2006
Posts: 4,136
Quote:
Now you are just trolling and so we have probably reach the I don't give a rats what you believe point and exit.


charge and motion !


3/4 inch of dust build up on the moon in 4.527 billion years,LOL and QM is fantasy science.
paul #56270 07/15/16 01:56 PM
Joined: May 2011
Posts: 2,819
O
Megastar
Offline
Megastar
O
Joined: May 2011
Posts: 2,819
Correct which is why you were closer than Bill G originally .. I was actually shocked he didn't know the answer.

If you haven't worked it out the emission shape is basically the average energy of the movement speed vibration of the charged particles. That holds for any particle(s) with or without electrons and orbitals.

This really is not hard most children can understand it and it is taught these days to 12 year olds.

Now wave goodbye to me and emit an EM wave sign off smile

Take care Paul, may you be in heaven 10min before the devil knows you dead and all that.

Last edited by Orac; 07/15/16 02:09 PM.

I believe in "Evil, Bad, Ungodly fantasy science and maths", so I am undoubtedly wrong to you.
Orac #56271 07/15/16 02:44 PM
Joined: Mar 2006
Posts: 4,136
P
paul Offline OP
Megastar
OP Offline
Megastar
P
Joined: Mar 2006
Posts: 4,136
one more thing before you slip away behind the curtain
Mr Wizzard.

Quote:
If you haven't worked it out the emission shape is basically the average energy of the movement speed vibration of the charged particles. That holds for any particle(s) with or without electrons and orbitals.


what we are told about a photon emission and the way it
is depicted is as if it were a tiny object that is emitted
away from an atom in a direction.

but we know that is not the case because a photon is emitted
in all directions like a expanding hollow sphere...

I added hollow just now because a single photon expands
away from its emitting atom.

so in the above when you state that ...

Quote:
the emission shape is basically the average energy of the movement speed vibration of the charged particles.


and this does not mean that the emission shape would be
different in front of its movement , to the side of its
movement , to the rear of its movement , top , bottom
... etc

the emission is bound to the average energy of the sum of
all motion of the particle as it is emitted.

in that there is no deformation of the emission as the
emission is formed and emitted due to any motion of the
particle.













3/4 inch of dust build up on the moon in 4.527 billion years,LOL and QM is fantasy science.
paul #56272 07/16/16 02:44 AM
Joined: May 2011
Posts: 2,819
O
Megastar
Offline
Megastar
O
Joined: May 2011
Posts: 2,819
Yes it has doppler shift applied, obviously at normal speeds its very very low and you can't measure it. From a fast moving sun in another galaxy it's very easy to detect and you know it as cosmological red shift. There is a slight hand wave in that answer in that classical doppler differs slightly from relativistic doppler but for the example you are using they will both give the same answer. However I suspect you are talking about the microscopic doppler of you standing near the atom.

Now if you were really thinking you would realize there is also a recoil of the atom as it emits the photon. As the atom is charged it must therefore produced a thermal emission and it does and has a name of recoil temperature or sometimes photon recoil heating
https://en.wikipedia.org/wiki/Recoil_temperature

As given in the article it's the lower limit for laser cooling and is very small under 1uK (1 micro Kelvin), you need tricks to go lower.

Small as the recoil may be it has been directly measured many hundreds of different ways in efforts to push laser cooling as it had to be understood correctly. Google "recoil energy direct measurement" will get you any number of the experiments.

You may get some interest looking at the machines used which are called reaction microscopes. They can do single photons as well as other more complex kinematic processes
Reaction microscope: https://www.mpi-hd.mpg.de/pfeifer/page.php?id=126

So the orbital emission case that everyone on the forum seems hell bent on attributing everything too also makes a thermal emission which will catch them out completely. So when we say orbital emission is only one frequency we overlook this based really on the magnitude difference between the two emissions much like we often overlook friction it depends with how much accuracy you want to apply to the discussion. At this limit we really are splitting hairs these emission play no real role in anything that isn't down near the quantum limit.

Last edited by Orac; 07/16/16 04:58 AM.

I believe in "Evil, Bad, Ungodly fantasy science and maths", so I am undoubtedly wrong to you.
Orac #56275 07/16/16 05:45 AM
Joined: May 2011
Posts: 2,819
O
Megastar
Offline
Megastar
O
Joined: May 2011
Posts: 2,819
There is a very complicated extension of your discussion that is very topical right this minute. I am not sure how much you will get but it is interesting.

Gavin Salam gave a talk which Tommaso Dorigo has done a write up on called strangely "How Much Light Does A Proton Contain"
http://www.science20.com/a_quantum_diaries_survivor/how_much_light_does_a_proton_contain-176396

It's topical because it runs around what we were discussing of charge + motion = em emission.

Quote:
Of course, the proton contains photons, as in its interior there are moving electric charges (the valence and sea quarks), and the acceleration of electric charges always involves photon emission.

Do you see what they are saying there must be what we would called photons internal to the proton ... which is rather weird to think about. We don't often think about photons being inside things, especially things like protons.

In the discussion he calculated the amount of photon energy in the proton and it won't come outside the proton which will get complicated to explain but the amount is not insignificant.
Quote:
So now we know with very good accuracy how much light do protons contain! If you think about it, it's awesome: we are made of protons, and protons are, in some part, made of light... And now we know how much of it.

See the physics is consistent ... charge + movement = EM emission ... even inside a proton smile

I probably should rephrase movement to acceleration but I will ignore that detail for now.

Even stranger think about how many protons are in your body and therefore how many photons you have inside you. Now I bags not breaking the news to the orbital believers ... nah they won't get it anyhow laugh

First principles is always fun and throws up things that will catch even seasoned scientists out because it is a different way of looking at a problem. That one will catch most average scientists out and I suspect there will be a few jokes created around it.

Last edited by Orac; 07/16/16 06:27 AM.

I believe in "Evil, Bad, Ungodly fantasy science and maths", so I am undoubtedly wrong to you.
Orac #56276 07/17/16 02:14 AM
Joined: Mar 2006
Posts: 4,136
P
paul Offline OP
Megastar
OP Offline
Megastar
P
Joined: Mar 2006
Posts: 4,136
Quote:
Now if you were really thinking you would realize there is also a recoil of the atom as it emits the photon. As the atom is charged it must therefore produced a thermal emission and it does and has a name of recoil temperature or sometimes photon recoil heating


the word recoil suggest that the atom would be moved in
a direction by a recoil due to photon emission.

since a photon emission is more of a expanding hollow sphere
and without using the word orbit then the terminology should
be changed to implosion temperature or implosion heating.

the recoil sounds more like a photon being emitted in a
direction causing the atom to recoil in the opposite
direction of photon emission.

and since a photon is emitted in all directions then
the atom should not feel any direction change due to photon emission.




3/4 inch of dust build up on the moon in 4.527 billion years,LOL and QM is fantasy science.
Orac #56278 07/19/16 05:44 AM
Joined: Sep 2012
Posts: 1,209
N
Megastar
Offline
Megastar
N
Joined: Sep 2012
Posts: 1,209
Originally Posted By: Orac
Paul I should say your last answer doesn't involve orbitals so it is at least better than the responses from Sam and Bill G. Why do the dam orbitals keep coming up people, give it up.

It's not hard it's pretty much explained in one paragraph in the thermal radiation page lets highlight the key bits
https://en.wikipedia.org/wiki/Thermal_radiation
Quote:
Thermal radiation is the emission of electromagnetic waves from all matter that has a temperature greater than absolute zero.[3] It represents a conversion of thermal energy into electromagnetic energy. Thermal energy consists of the kinetic energy of random movements of atoms and molecules in matter. All matter with a temperature by definition is composed of particles which have kinetic energy, and which interact with each other. These atoms and molecules are composed of charged particles, i.e., protons and electrons, and kinetic interactions among matter particles result in charge-acceleration and dipole-oscillation. This results in the electrodynamic generation of coupled electric and magnetic fields, resulting in the emission of photons, radiating energy away from the body through its surface boundary. Electromagnetic radiation, including light, does not require the presence of matter to propagate and travels in the vacuum of space infinitely far if unobstructed.


It's pretty dam simple even a layman should be able to understand it yet somehow the layman keep bringing us back to the stupid atomic orbitals.

Single charged particles clumped together will give off thermal emission and they don't even have any orbitals ... it has nothing to do with orbitals.

CHARGE PARTICLE + MOVEMENT is the answer for goodness sake it's not rocket science people. I have never seen layman make such a simple thing so hard, have none of you ever moved a magnet near a wire at school and measured what happens?

If I see one more post about thermal emission that includes the word orbital or electrons, I think I will request AR2 to ban the person for gross stupidity.

The thermal emission is because lots of charged particles are jiggling around being accelerated and decelerated and because of that they emit EM. You make them hot they jiggle more and emit more .... NOT HARD PEOPLE.

Funny conclusion to that if you are smart is waving your hand gives off EM smile
Someone even got it right here ... something that would never happen on this forum.
http://physics.stackexchange.com/questions/200217/can-i-produce-radio-waves-by-waving-my-hand

So the take home lesson is there are hundreds of ways to produce EM and very few involve orbitals.

END OF STORY.


TO ABOVE WE MUST ADD ONE MORE FACT
CZERENKOW's RADIATION

Each Collision with EM wave is able be SOURCE for other lower or more higher freg. (Hz)

em radiation)))...........m >>> motion ........((( em radiation


please hit mass m by wave (back) and (front ) You will generate
other Hz

DOUBLE SLITS ???

HOW EM wave is changing electron's motion ?
Em wave is faster than electron !!!




EM wave IS PUSHING ELECTRON !!!

1861 J. C. Maxwell, published his theory of electromagnetic fields and radiation,which shows that light has momentum and thus can exert pressure on objects.

Exist also one more problem EM wave is giving to elecron (mass m )
special rotation



Earth is making rotation because exist INVERSE SQUARE LOW !!!
ENERGY's density near SUN is more HIGH


During double slits experiment electron is changing own motion and it starting make rotation respect to own axis !



Last edited by newton; 07/19/16 05:50 AM.
newton #56279 07/19/16 06:48 AM
Joined: Sep 2012
Posts: 1,209
N
Megastar
Offline
Megastar
N
Joined: Sep 2012
Posts: 1,209
MOTION is GENERATING waves !!!

Wave that is generating by motion can not be describe BY PLANCK's constant !!!!

THEORY
1861 J. C. Maxwell, published his theory of electromagnetic fields and radiation,which shows that light has momentum and thus can exert pressure on objects.

2011 Grover Swartzlander first began to examine a revolutionary concept in optical physics after studying the flight of a moth. He watched the animal use its wings to create lift..... ( find more in google)

2012 Marosz



BEFORE !!! collision with plasticyne photon's energy E1
after collision we have E2+E3 energy !!!

E2 - kinetic energy ( Plasticyne )
E3- Energy that we lost during collision !!!


paul #56280 07/19/16 07:38 AM
Joined: Sep 2012
Posts: 1,209
N
Megastar
Offline
Megastar
N
Joined: Sep 2012
Posts: 1,209



Page 1 of 5 1 2 3 4 5

Link Copied to Clipboard
Newest Members
debbieevans, bkhj, jackk, Johnmattison, RacerGT
865 Registered Users
Sponsor

Science a GoGo's Home Page | Terms of Use | Privacy Policy | Contact UsokÂþ»­¾W
Features | News | Books | Physics | Space | Climate Change | Health | Technology | Natural World

Copyright © 1998 - 2016 Science a GoGo and its licensors. All rights reserved.

Powered by UBB.threads™ PHP Forum Software 7.7.5